Practice Test


Q1) The word Adivasi was used for the first time to refer to the tribal people by;
Show Answer


Q2) Which among the following is popularly known as Gadkari Movement? Show Answer


Q3) Consider the following statements
1. Pagal Panthis were semi-religious sect founded by Karam Shah.
2. Pagal Panthis were advocated radical, religious, social and political changes.
3. Saiyed Ahmed was the founder of Faraizi Movement.
Select the correct answer using the codes given below
Show Answer


Q4) Consider the following statements
1. The Santhal hool was the most massive among the various tribal revolts.
2. The Santhals lived in Daman-i-koh area.
3. Kanhu was hanged publicly and Sidhu was never caught.
Select the correct answer using the codes given below
Show Answer


Q5) Which of the following revolts occurred in Bengal and Eastern India?
1. Gadkari Revolt
2. Munda Revolt
3. Wahabi Movement
4. Kittur Movement
5. Kuka Rebellion
6. Kol Rebellion
7. Koli Revolt
Select the correct answer using the codes given below

Show Answer


Q6) Select the correct answer from the codes given about the Chronological sequence of the movements.
1. Kuka Movement
2. Wahabi Movement
3. Mundas Revolt
4. Mopla Revolt
Codes Show Answer


Q7) What was common between the Wahabi and the Kuka Movement?
1. Hatred against butchers.
2. Religious fanaticism.
3. Use of violent methods.
Codes Show Answer


Q8) Consider the following statements in the context of Lord Lyton
1. Lord Lyton recommended the Thomsonian system of vernacular education for whole of the North-Western Provinces.
2. He pursued free trade and abolished duties on 29 British manufactured goods which accelerated drain of wealth of India.
3. He appointed a Police Commission under Andrew Frazer.
Which of the statements given above is/are correct? Show Answer


Q9) Consider the following events and select which one of them took place during the tenure of Lord Dalhousie
1. Second-Anglo Sikh War
2. Widow Remarriage Act
3. Abolished Slavery
4. Annexation of Mysore
5.Abolished Titles and Pensions
6. Treaty of Amritsar
7. The Indigo Riots in Bengal
Select the correct answer using the codes given below
Show Answer


Q10) Assertion (A) Ilbert BIll proposed trial of British subjects could be held by Indian Magistrates
Reason (R) The Indian Magistrates felt humiliated on being considered ill fit to held a trial of a British subject Show Answer


Q11) Consider the following statements in the context of Warren Hastings
1. Hastings played crucial role in the foundation of Asiatic Society of Bengal
2. Hastings was impeached on his return to England.
3. The Third Mysore War took place in the tenure of Warren Hastings
Which of the statements given above is/are correct? Show Answer


Q12) Who said, "English education would make the Indian people gladly accept the British rule? Show Answer


Q13) Assertion (A) The Wardha Scheme of 1937 was related with basic education.
Reason (R) The Scheme was worked out by Zakir Hussain Committee, to review the failing standards of English education among Indian students.
Codes Show Answer


Q14) Which document became the basis of Indian University Act of 1904? Show Answer


Q15) Consider the following statements in the context of Wardha Scheme.
1. The Scheme emphasised on manual work.
2. The Scheme attached little importance of mother tongue and much to English
Select the correct answer using the codes given below
Show Answer


Q16) Consider the following statements
1. Wood's Despatch called as Magna Carta of Education in India.
2. Three universities in the presidency towns were established on the recommendation of Hunter Commission.
3. Sadller Commission was primarly appointed to review on Bombay University.
4. University Grants Commission (UGC) was constituted based on the recommendation of Radhakrishnan Committee.
Which among the following are correct? Show Answer


Q17) The educated middle class in India : Show Answer


Q18) His principle forte was social and religious reform. He relied upon legislation to do away with social ills and work unceasingly for the eradication of child marriage and the purdah system. To encourage consideration of social problems on a national scale, he inaugurated the Indian national social Conference, which for many years met for its annual session alongside the Indian National Congress. The reference in this passage is to : Show Answer


Q19) Which one of the following is an imortant historical novel written during the letter half of the nineteenth century? Show Answer


Q20) Which one of the following place did Kunwar Singh, a prominent leader of the Revolt of 1857 belong to ? Show Answer


Q21) Which one of the following territories was not affected by the Revolt of 1857 ? Show Answer


Q22) With reference to the 'revolt of the year' who of the following was betrayed by 'friend' captured and put to death by the British? Show Answer


Q23) Who was the Governor-General of India during the Sepoy Mutiny? Show Answer


Q24) In 1930, Mahatma Gandhi started civil Disobedience Movement from : Show Answer


Q25) The meeting of India and British political leaders during 1930-32 in London has often been referred to as the First., Second and Third Round Table Conference. It would be incorrect to refer to them as such because : Show Answer


Q26) Which one of the following is not correct about the Cabinet Mission plan?
Show Answer


Q27) Who among the following suggested the winding up of the Indian National Congress after India attained independence? Show Answer


Q28) The Indian Muslim, in general ,were not attracted to the Extremist movement because of ; Show Answer


Q29) Who was the leader of the Gadar Party? Show Answer


Q30) Lord Mountbatten came to India as a Viceroy with specific instruction to : Show Answer


Q31) 'Abinava Bharat' a secret society of revolutionaries was organised by : Show Answer


Q32) The first venture of Gandhi in all-India political was the: Show Answer


Q33) As an alternative to the partition of India, Ghandhiji suggested to Mountbatten that he ; Show Answer


Q34) The native state of Tripura became involved in the freedom movement early in the 20th century because ; Show Answer


Q35) After returning from south Africa, Gandhiji launched his first successful satyagraha in ; Show Answer


Q36) During the Indian freedom struggle, the Khudai Khidmatgars, also known as Red Shirts called for: Show Answer


Q37) The last opportunity to avoid the partition of india was lost with the rejection of : Show Answer


Q38) During the indian freedom struggle, who among the following proposed that Swaraj should be defined as complete independence free from all foreign control?
Show Answer


Q39) During the freedom struggle, Aruna Asaf Ali was a major woman organizer of underground activity in : Show Answer


Q40) Which one of the following began with the Dandi March? Show Answer


Q41) What was the immediate cause for the launch of the Swadeshi movement ? Show Answer


Q42) With reference to indian Freedom struggle, Uash Mehta is well-known for Show Answer


Q43) The Congress ministries resigned in the seven provinces in 1939,because Show Answer


Q44) During Indian freedom struggle, the National Social Conference was formed. What was the reason for its formation? Show Answer


Q45) After resigning from Indian National Congress Subhash Chandra founded : Show Answer


Q46) Which one of the following chronological orders of the given events is correct ? Show Answer


Q47) Which one of the following is the correct chronological sequence of events during Indian freedom struggle ? Show Answer


Q48) Which one of the following is the correct chronological sequence of events during the British rule in India? Show Answer


Q49) Who among the following was the founder of the Servants of India society ? Show Answer


Q50) Disciples of which one of the leaders in Bengal during the Indian freedom struggle were called Young Bengal ? Show Answer


Q51) Who among the following established Fegusson College at Pune in the Year 1885 ? Show Answer


Q52) The war of Independence of 1857 failed because : Show Answer


Q53) The main effects of the revolt of 1857 was that it Show Answer


Q54) In which state, the kondo Dora revolt broke out ? Show Answer


Q55) Before quit India movement, the British government made a plan to imprison Gandhi and the other Congress leaders. The plan was known as - Show Answer


Q56) Who started the Ahmadia Movement in 1889? Show Answer


Q57) Who is related to Untouchable movement ? Show Answer


Q58) To free Goa from Portuguese subjection. Who started Civil Disobedience movement ? Show Answer


Q59) Who was the first female Chairman of Indian National Congress? Show Answer


Q60) Who was a famous Hindu leader and a member of Hindu Mahasabha and also the president of Trade Union Congress? Show Answer


Q61) Rabindra Nath Tagore who had been awarded the nobel prize,returned his knighthood, to British Government in protest against which event? Show Answer


Q62) During the Partition of India which was the province of British India to propose the plan of united and autonomous existence? Show Answer


Q63) During whose tenure, the resolution of total independence was passed? Show Answer


Q64) Who granted freedom to India? Show Answer


Q65) Under the dual government in Bengal, the administrations was carried on by - Show Answer


Q66) As per the Act of 1919, which of the statement was not correct ? Show Answer


Q67) By a regulation in 1793, the district collector was deprived of his judicial powers and made the collecting agent only. What was the reason for such regulation? Show Answer


Q68) Which of the following provision was not made in the Charter Act of 1833? Show Answer


Q69) Consider the following statement regarding the proposals of cripp's mission -
1) A constitution making body consisting of the elected representative from British India and members from the princely state
2) India was promised to have dominion status.
3) It suggested partition of India.
4) Setting up an executive council composed of Indian alone.
Which of the statement is/are correct ? Show Answer


Q70) What were the causes of the adoption of the Quit India Resolution ? Consider the statements -
1) Gandhiji felt the moral of the people had to be built up to resist any impending Japanese invasion.
2) The cripps mission failed to meet nationalists demands.
3) The congress at last realized that the Allies cause was an imperialist cause alone.
4) It was felt that any further inactivity would be tantamount to accepting the right of the British government to decide India's fate without considering the indian's wishes.
Which of these statements is/are correct? Show Answer


Q71) Write the correct chronological order of the following -
1) Foundation of swaraj party
2) Jallianwala bagh tragedy
3) Congress league pact
4) Chauri-Chura incident Show Answer


Q72) Arrange the following social religious reformers in the Chronological order of their birth -
1) Debendranath Tagore
2) Keshavchandra Sen
3) Ramakrishna Paramshana
4) Rama Mahan Roy
5) Swami Vivekanand
6)Swami Dayanand' Show Answer


Q73) Which of the following statement regarding Muhammad Ali Jinnah is not true ?
1) He became a communal nationalist in 1913 when he joined the muslim league.
2) He and Tilak were the joint authors of Lucknow Pact
3) In 1919-20, he supported Gandhi's non cooperation movement.
4) He played important role on Calcutta session of Congress in 1906.
Show Answer


Q74) With reference to the Second Round Table Conference, consider the following statement -
1) Viceroy Irwin agreed to release all prisoners who were imprisoned without any guilty.
2) The provision was given for provincial autonomy.
Which of these statement is/are correct ? Show Answer


Q75) The revolt of 1857 became a failure because?
1) The lack of coordination among the force fighting in different regions.
2) Most of the Indian rulers did not participate.
3) There was no effective centralized leadership.
4) The rebels lacked modern armory. Show Answer


Q76) Which of the following was associated with Lord Lytton?
1) The title Kaiser-i-hind was conferred on Queen Victoria.
2) Censorship was imposed on vernacular press
3) The factory Act was passed.
4) Quetta was occupied by the British. Show Answer


Q77) The Sard Act, 1930 provided?
1) Civil marriage
2) Penalisation for bigamy
3) Penalisation of parties to a marriage in which the girl was below 14 years and the boy was below 18 years of age
4) Legalization of inter-cast marriage.
Show Answer


Q78) The Indians decided to boycott the Simon commission appointed to look into working of the reforms introduced under the Act of 1919 because -
1) in intended to provide communal representation to the untouchables.
2) it denied the congress the right to nominate a muslim as a representative on the commission.
3) it did not include any Indians as its member.
4) it accorded greater representation to the muslims that their numerical strength warranted in muslim minority areas.
Which of these statements is/are correct? Show Answer


Q79) With reference to the British rule in India, consider the following statements
1. Raja Ram Mohan Roy's Miratul Akbar had to stop being published as a consequence of John Adam's Licensing Regulations, 1823.
2. The Censorship of press Act, 1799 required the publisher to submit all material for pre-censorship to the Secretary to the Government.
Which of the statements given above is/are correct? Show Answer


Q80) Consider the following statement -
1) Balwantrai Mehta was one of the person chiefly responsible for the initiative to convene the All India states people's conference in 1927
2)C.Rajagopalchari was chiefly responsible in persuading Mahatma Gandhi to decide to initiate the 'Individual Satyagrah'
Which of the above statements is/are correct? Show Answer


Q81) During the Indian Freedom Struggle, who of the following started the weekly Yugantar in April, 1906? Show Answer


Q82) Consider the following statement -
The Regulating Act of 1773 sought to
1) reform the constitution of the East India company.
2) provide remedies against the illegalities and oppression of the company's servants in India.
Which of the above statements is/are correct? Show Answer


Q83) Which of the following newspapers advocated revolutionary terrorism during the period of the Indian Freedom Struggle?
1. Sandhya
2. Bande Matram
3. Yugantar
4. Ghulam Giri
5. Kaal
6. Prabudha Bharat
Choose the correct answer using the codes given below
Show Answer


Q84) Which of the following statements is incorrect?
Show Answer


Q85) Consider the following statements -
1) The discussion in the third round table conference eventually led to the passing of India Act of 1935.
2) The government of India Act of 1935 provided for the establishment of All India Federation to be based on a Union of the provinces of British India and the Princely state.
Which of these statements is/are correct ?
Show Answer


Q86) Consider the following statements -
1) The First session of the Indian National Congress was held in Calcutta.
2) The second session of the Indian National Congress was held under the presidentship of Dadabhai Naoroji.
3)Both Indian National congress and muslims league held their sessions at lucknow in 1916 and concluded the Lucknow pact.
which of these statement is/are correct ?
Show Answer


Q87) Consider the following Viceroys of India during the British rule.
1) Lord Curzon
2) Lord Chelmsford
3) Lord Harding
4) Lord Irwin
Arrange it in chronological order - Show Answer


Q88) With reference to Indian freedom struggle, consider the following statements -
1) C.R Das and Matilal Nehru formed the Congress Khilafat Swarajaya Party.
2) In 1919 , Gandhiji was elected President of the Khilafat conference.
3) The communist Party of India was banned by the Government in 1934
Which of these statement is/are correct ? Show Answer


Q89) Which of the following statement is/are correct regarding Brahmo Samaj?
1) It opposed idolatry
2) It denied the need for a priestly class for for interpreting the religious texts.
3) It popularized the doctrine that the Vedas are infallible.
Select the correct answer using the codes given below : Show Answer


Q90) Consider the following statement -
The Non-cooperation movement led to :
1) Congress becoming a mass movement for the first time
2) Growth of Hindu-Muslim unity
3) Removal of fear of the British might from the minds of the people
4) British government's willingness to grant political concession to Indians
Of these statement : Show Answer


Q91) What is the correct sequence of the following events?
1) The August offer
2) The I.N.A trail
3) The Quit India Movement
4) The Royal Indian Naval Ratings revolt
Select the correct answer using the codes given below : Show Answer


Q92) Consider the following statement with regard to Champaran Satyagraha (1917)
1) The Champaran Satyagraha marked mahatma Gandhi's first appearance in India politics as a leader of the masses .
2) Young nationalist like Rajendra Prasad and J.B Kriplani and worked with Mahatma Gandhi at Champaran
3) It was during this movement that Mahatma Gandhi formulated his philosophy of Satyagraha.
Which of the statements given above is/are correct ?
Show Answer


Q93) Consider the following events :
1) Indigo Revolt
2) Santhal Rebellion
3) Deccan Riot
4) Mutiny of the Sepoys
The correct chronological sequence of these events is :
Show Answer


Q94) Assertion : The Quit India movement of 1942 marked the culmination of the Indian freedom struggle.
Reason : After the quit India movement it was only a matter of time before the mechanics of transfer of power was determined. Show Answer


Q95) Assertion : Montague stated that the British government in India would associate Indians in every branch of the administration of India.
Reason : The British government thought that self governing institution would have to be developed in India with a view to the realisations of responsible government in India within the British empire. Show Answer


Q96) Assertion : In the provinces, under the system of Dyarchy, the ministers had to depend upon the official bloc.
Reason : In the provinces, there were no strong political parties to back the ministers. Show Answer


Q97) Assertion : The Lahore session of the congress declared 'Complete Independence' as the aim of congress.
Reason : Jawaharlal Nehru was the President of the Lahore session of the congress. Show Answer


Q98) Assertion : The Indians Councils Act 1892 provided separate representation for Muslims in the Central and Provincial Legislative councils.
Reason : Dyarchy in the provinces was introduced by the Indian council Act ,1919 Show Answer


Q99) Assertion : The non-cooperation movement was called off by the congress Working committee.
Reason : A mob violence occurred at Chauri-Chaura in 1922
Show Answer


Q100) Assertion : The Khilafat movement did bring the urban muslims into the fold of the National movement.
Reason : There was a predominant element of anti-imperialism in both the National and Khilafat movement Show Answer


Q101) Assertion : Gandhi stopped the Non-cooperation movement in 1922
Reason : Violence at Chauri-Chaura led him to stop the movement Show Answer


Q102) The Vernacular Press Act, 1878 came against
1. Mirat-ul-Akbar
2. The Som Prakash
3. The Bharat Mihir
4. The Dacca Prakash
5. The Sahachar
6. The Madras Courier
Select the correct answer using the codes given below Show Answer


Q103) Which one of the following historians has written about the nature of the Movement of 1857 that "It was neither first nor national war of independence."? Show Answer


Q104) Consider the following statements about Revolt of 1857
1. The Infamous Massacre related with Nana Saheb took place at Bibigarh, Kanpur.
2. VD Sarvakar was the first person to opine Revolt of 1857 as the first Indian War of Independence.
3. Major General Havelock defeated the forces of Nana Saheb.
4. The revolt came to an end with the capture of Tatya Tope.
Which of the statements given above is/are correct?
Show Answer


Q105) Which one among the following statements is correct?
Show Answer


Q106) Which one of the following do you think in the most satisfactory way of explaining the cause of the meeting in 1857? Show Answer


Q107) The Arya Samajists split on the issue of
Show Answer


Q108) According to the Dayananda, which of the following was full of false teachings? Show Answer


Q109) Consider the following statements of Arya Samaj Movement?
1. Dayanand Saraswati rejected later religious thoughts, which were in conflict with the Vedas.
2. The Arya Samajists fought against untouchability.
3. The Arya Samaj Movement tried to prevent the conversion of Hindus to other religions, leading to inter-religious tension in India in the 20th Century.
Which of the statements given above are correct? Show Answer


Q110) Which of the statements pairs is not correctly matched Show Answer


Q111) Which one of the following was not a demand of the Prarthana Samaj? Show Answer


Q112) Consider the following statements
1. Deoband Movement was founded by Muhammed Qasim Nanaotavi and Qasim Ahmed Gangohi.
2. The objective of the movement was to improve the spiritual and moral condition of Indian Muslims.
Which of the statements given above is/are correct? Show Answer


Q113) With regard to Singh Sabha Movement which of the statements is/are correct? Show Answer


Q114) Consider the following statements
1. Raja Ram Mohan Roy was an advocate of ethical reform and political liberalism in India.
2. He was the founder of Benaras Hindu University.
3. He attacked the practice of Sati.
Which of the statements given above is/are correct? Show Answer


Q115) Which of the following statement(s) is/are correct?
1. Suddhi Movement under the Ramakrishan Mission bring back the Hindu converted to other religions.
2. Arya Samaj played a progressive role in furthering the cause of social reforms and spread of education in South India.
Codes Show Answer


Q116) Which of the statements given below about Vivekanand are correct?
1. He considered the veda to be infallible.
2. He believed that vedanta was fully rational.
3. He condemned the caste system.
Codes Show Answer


Q117) Consider the following statements
1. Prarthana Samaj believed that true love of God lay in the service of its children without any social or religious distinction.
2. Prarthana Samaj rejected Vedas or Upanishads.
3. Prathana Samaj paid more emphases on Bhakti (devotion).
Which of the above statements about Prarthana Samaj is/are correct? Show Answer


Q118) Which of the following books written by Raja Ram Mohan Roy?
1. Gift of Monotheist
2. Mira-ul-Akbar
3. Precepts of Jesus
4. Tuhfat-ul- Muwahhidin
5. Veda Bhashya
Codes Show Answer


Q119) Consider the following statements
1. Christian missionaries arriving to India, with the intention to turn a majority of population into Christians.
2. The Christian missionaries tried to cast Christianity in the light of a better religion and with economic inducements convinced the poor Indians into Christianity.
Which of the above statement(s) is/are correct? Show Answer


Q120) With reference to the period of colonial rule in India, Home Charges formed an important part of drain of wealth from India, Which of the following funds constituted Home Charges?
1. Funds used to support the India Office in London.
2. Funds used to pay waging wars outside India by the British.
Select the correct answer using the codes given below Show Answer


Q121) Which one of the factors given below did not help the moneylenders to exploit the cultivation under the British rule in India? Show Answer


Q122) Which of the following statements is/are correct? Show Answer


Q123) Consider the following statements
1. The Champaran Satyagraha marked Gandhi's second appearance in Indian Politics as a leader of the masses.
2. The Champaran Satyagraha was launched to address the problems faced by Indigo Plantation Workers.
Which of the statements given above is/are correct? Show Answer


Q124) Through which principle/device did Mahatma Gandhi strive to bridge economic inequalities? Show Answer


Q125) What was the immediate cause for the launch of the Swadeshi Movement? Show Answer


Q126) The core of the Safety-Valve thesis rests on the assumption that Show Answer


Q127) Consider the following statements about Madam Bhikaji
1. Madam Cama unfurled the National Flag at the International socialist Conference in Stutguard Germany in the year 1907.
2. Madam Cama served as private Secretary to Dadabhai Naoroji.
3. Madam Cama was born to Parsi Parents.
Which of the statements given above is/are correct? Show Answer


Q128) Who of the following was the first President of the Indian Trade Union Congress in 1920? Show Answer


Q129) In October, 1920, who of the following headed a group of Indians gathered at Tashkand to set up a Communist Party of India? Show Answer


Q130) Which party was founded by SC Bose in the year 1939 after he broke away from the Congress? Show Answer


Q131) Assertion (A) a split in AITUC took place in 1929.
Reason (R) split became inevitable, as there was growing influence of Communists in the AITUC. Show Answer


Q132) Which among the following about Mahatma Gandhi Non-Cooperation Movement are correct?
1. Refusal to attend Government Durbars and official functions.
2. Participation in elections.
3. Participation in rallies for the boycott of foreign goods.
4. Surrender of titles.
Select the correct answer using the codes given below Show Answer


Q133) Consider the following statements about the Non-Cooperation Movement
1. Gandhiji's appeal for Non-Cooperation Movement had attracted both the moderate and the extremist rank and file, for he had cleverly combined the goal of the former, Swaraj within the empire, with the means of the later Non-Cooperation
2. CR Das decided to enter the Legislative Council not to help the British, but to embarrass them. It was to be a form of Non-Cooperation from within.
Which of the statement(s) given above is/are correct? Show Answer


Q134) Consider the following statements in regard to Khilafat Movement?
1. Khilafat Movement, an organisation that wanted to protect the position of the Ottoman Sultan as leader of all Muslims.
2. The Khilafat Committee was formed in Bombay, in March, 1919, to defend the power of Khalifa.
3. Brother Mohammad Ali and Shaukat Ali were two active Khilafat leaders.
Which of the statements given above is/are correct? Show Answer


Q135) With reference to the period of Indian freedom struggle, which of the following was/were recommended by Nehru Report?
1. Complete Independence for India.
2. Joint electorates for reservation of seats for minorities.
3. Provision of Fundamental Rights for the people of India in the Constitution.
Select the correct answer using the codes given below Show Answer


Q136) The Swaraj Party decided to take part in the Reforms Scheme under the Government of India Act, 1919 because
Show Answer


Q137) At which Congress Session was the working committee authorised to launch a programme of Civil Disobedience? Show Answer


Q138) The Lahore Session of the Indian National Congress (1929) is very important in history, because
1. The Congress passed a resolution demanding complete independence.
2. The rift between the extremists and moderates was resolved in that session.
3. A resolution was passed rejecting the two-nation theory in that session.
Which of the statements given above is/are correct? Show Answer


Q139) Which one of the following observation is not true about the Quit India Movement of 1942? Show Answer


Q140) With reference to Indian Freedom struggle Usha Mehta is well known for
Show Answer


Q141) During the freedom struggle, Aruna Asif Ali was a major woman organiser of underground activity in Show Answer


Q142) Assertion (A) The result of the elections of 1937 was an eye opener to Muslim League.
Reason (R) The League captured majority of the seats reserved for Muslims and this created pressure upon the leadership to take the Pakistan movement further. Show Answer


Q143) Who, among the following, was not associated with the formation of 'UP Kisan Sabha' in February, 1918? Show Answer


Q144) Consider the following statements
1. Result of local grievances.
2. Leaderships of these uprisings were semi-feudal in character.
3. Backward looking and traditional in outlook.
4. They were localised and isolated.
Which of the above were the weakness of the civil rebellions, tribal revolts and peasant movements? Show Answer


Q145) After Quit India Movement, C Rajagopalachari issued a pamphlet entitled
'The way out'. Which one of the following was a proposal in this pamphlet? Show Answer


Q146) Consider the following statements about Jawaharlal Nehru
1. He was the President of the Congress Party in 1947.
2. He presided over the Constituent Assembly.
3. He formed the first Congress Ministry in United Province before India's Independence
Which of the statements given above is/are correct? Show Answer


Q147) The members of the Constituent Assembly, which drafted the Constitution of India were Show Answer


Q148) Mahatma Gandhi undertook fast unto death in 1932, mainly because Show Answer


Q149) Consider the following statements
The most effective contribution made by Dadabhai Naoroji to the cause of Indian National Movement was that he
1. Exposed the economic exploitation of India by the British.
2. Interpreted the ancient Indian texts and restored the self-confidence of Indians.
3. Stressed the need for eradication of all the social evils before anything else.
Which of the statement(s) given above is/are correct? Show Answer


Q150) The Lahore Session of the Indian National Congress (1929) is very important in history because
1. The Congress passed a resolution demanding complete independence.
2. The rift between he extremists and moderates was resolved in that session.
3. A resolution was passed rejecting the two-nation theory in that session.
Which of the statement(s) given above is/are correct? Show Answer


Q151) What was the reason for Mahatma Gandhi to organise a Satyagraha on behalf of the peasants of Kheda?
1. The Administration did not suspend the land revenue collection in spite of a drought.
2. The administration proposed to introduce Permanent Settlement in Gujarat.
Which of the statements given above is/are correct? Show Answer


Q152) For the Karachi Session of Indian National Congress in 1931, presided over by Sardar Patel, who drafted the Resolution on Fundamental Rights and Economic Programme? Show Answer


Q153) Consider the following statements
1. The Bombay Manifesto signed in 1936 openly opposed the preaching of socialist ideals.
2. It evoked support from a large section of business community from all across India.
Which of the statements given above is/are correct? Show Answer


Q154) Consider the following statements about the administrative changes after 1857
1. Hostility to educated Indians.
2. Landlords act as counterweights to the nationalists.
3. Social services ignored.
4. Reversal of policy of support to social reforms.
5. Racial arrogance.
Select the correct answer using the codes given below Show Answer


Q155) Which of the following was/were the principle features of the Cabinet Mission Plan
1. Rejection of Pakistan.
2. Provinces to have autonomy and residual powers.
3. Common centre for defence and external affairs only.
Select the correct answer using the codes given below Show Answer


Q156) What was the main aim of British Government's Policy after 1946? Show Answer


Q157) Consider the following statements regarding CR Formula of 1944.
1. Muslim League should support Independence of India.
2. In case of partition. common centre for defence.
3. Cooperate Interim Government.
Select the correct answer using the codes given below Show Answer


Q158) Which of the following were the causes for the Quit India Movement of 1942?
1. British provocation before the passage of the Quit India Resolution and their policy of wholesale repression after its passage.
2. The condemnable behaviour of most of the foreign toops stationed in India.
3. The popular mood caused by the rout of the British by Japan in south East Asia.
4. Shortages of certain essential commodities like rice and salt and shooting up of prices.
Select the correct answer using the codes given below Show Answer


Q159) Why Cripps Mission failed? Show Answer


Q160) Consider the following statements
1. Congress attended the First Round Table Conference.
2. After Gandhi-Irwin Pact of 1931, Congress agreed to attend the Second Round Table Conference.
3. Karachi Session of Congress (1931) endorsed the Gandhi-Irwin Pact.
Select the correct answer using the codes given below Show Answer


Q161) Consider the following emergence of new forces during the Indian National Movement.
1. Spread of socialist ideas
2. Caste movement
3. Growth of trade unionism
Which of the above forces emerged during the 1920's? Show Answer


Q162) Which of the following statements about the Trade Union Movement in 1938 are correct
1. AITUC was finally replaced by the Hindustan Mazdoor Sangh.
2. NTUF got itself affiliated to the AITUC.
3. Indian Trade Union Federation got emerged in the National Trades Union Federation (NTUF)
4. Red Trade Union Congress got merged into the All India trade Union Congress (AITUC)
Select the correct answer using the codes given below Show Answer


Q163) Consider the following statements
1. One-Third representation to Muslims in the Central Legislature.
2. Reservations to Muslims in Bengal and Punjab Legislatures proportionate to their population, till adult suffrage was established.
3. Residual powers to provinces.
which of the above was/were amendments proposed by Jinnah after Nehru Report's consideration? Show Answer


Q164) What was/were the impact of appointment of Simon Commission? Show Answer


Q165) Which of the following Government Acts was/were enacted for repression of Swadeshi Movement?
1. Seditious Meeting Act, 1907.
2. Criminal Law (Amendment) Act, 1908.
3. Indian Press Act, 1910
4. Explosive Substance Act, 1908
Select the correct answer using the codes given below
Show Answer


Q166) Consider the following statements
1. Netaji Subhash Chandra Bose proclaimed the establishment of a Provisional Government of Free India at Singapore.
2. The revolutionaries under the leadership of Surya Sen set up a Provision Independent Government of Independent Government of India at Chittagong.
Which of the statements given above is/are correct? Show Answer


Q167) Consider the following statements
1. Bal Gangadhar Tilak is most frequently associated with the national fight for the freedom of press.
2. Bal Gangadhar Tilak was the first to advocate bringing the lower middle classes, peasants, artisans and workers into the congress fold.
Select the correct answer using the codes given below Show Answer


Q168) The Drain Theory incorporated all threads of nationalist critique that it denuded India of its productive capital. According to nationalist estimates, the economic drain at that time was
1. More than the total land revenue.
2. Half of the total government revenue.
3. One third of the total savings.
Select the correct answer using the codes given below Show Answer


Q169) Which of the following parties were established by Dr BR Ambedkar?
1. The Peasants and Workers Party of India.
2. All India Scheduled Castes Federation.
3. The Independent Labour Party.
Select the correct answer using the codes given below Show Answer


Q170) The Rowlatt Act aimed at Show Answer


Q171) The distribution of powers between the Centre and the States in the Indian Constitution is based on the scheme provided in the Show Answer


Q172) The Congress ministers resigned in the seven provinces in 1939, because Show Answer


Q173) With reference to the Indian freedom struggle, Usha Mehta is well known for Show Answer


Q174) Which among the following provided a common factor for tribal insurrection in India in the 19th Century? Show Answer


Q175) Four resolutions were passed at the famous Calcutta Session of Indian National Congress in 1906. The question of either retention or of rejection of these four resolutions became the cause of a spin in Congress at the next Congress Session held in Surat in 1907.
Which one of the following was not one of those resolutions? Show Answer


Q176) Who among the following were official Congress negotiators with Cripps Mission? Show Answer


Q177) Consider the following statements
1. Dr Rajendra Prasad persuaded Mahatma Gandhi to come to Champaran to investigate the problem of peasants.
2. Acharya JB Kripalani was one of Mahatma Gandhi's colleagues in his Champaran investigations.
Which of the statements given above is/are correct? Show Answer


Q178) In the context of the Indian Freedom Struggle, 16th October, 1905 is well-known for which one of the following reasons? Show Answer


Q179) The Cripps' proposals include the provision for
1. Full independence for India
2. Creation of Constitution-making body
Which of the statement(s) given above is/are correct?
Show Answer


Q180) Which one of the following suggested the reconstruction of the Viceroys Executive Council in which all the portfolios including that of war members were to be held by the Indian leaders? Show Answer


Q181) Who among the following rejected the title of Knighthood and refused to accept a position in the Council of the Secretary of State for India? Show Answer


Q182) Assertion (A) The Congress Ministers in all the provinces resigned in the year 1939.
Reason (R) The Congress did not accept the decision of the Viceroy to declare war against Germany in the context of the World War II.
Codes Show Answer


Q183) Assertion (A) According to the Wavell Plan, the number of Hindu and Muslim members in the Executive Council were to be equal.
Reason (R) Wavell thought that this arrangement would have avoided the Partition on India.
Codes Show Answer


Q184) Assertion (A) In 1916, Maulana Mohammad Ali and Abdul Kalam Azad resigned from the Legislative Council.
Reason (R) The Rowlatt Act was passed by the Government in spite of being opposed by all Indian member of the Legislative Council.
Codes Show Answer


Q185) The First Factory Act for restricting the working hours of women and children and authorising local government to make necessary rules was adopted during whose time? Show Answer


Q186) Under whose presidency was the Lahore Session of the Indian Congress held in the year 1929, wherein a resolution was adopted to gain complete independence from the British? Show Answer


Q187) Certain measures of Dalhousie created serious discontent in India, which were also responsible for the Revolt of 1857. Which of the following measure was not one of them? Show Answer


Q188) The Khilafat Movement received support from both Hindu and Muslim and it was led from the front by Gandhiji. In spite of this, the movement lost momentum. Why? Show Answer


Q189) Which of the following statements Theosophical Society are correct?
1. Its founders were Non-Indians.
2. Its advocates were in favour of the revival of Hinduism.
3. It advocated for the adoption of rational and scientific approach based on Western ideas.
4. It helped people overcome the sense of false pride.
Select the correct answer using the codes given below Show Answer


Q190) Which of the following statements regarding Home Rule Leagues in India is correct?
1. The movement was led by moderate leaders of Congress.
2. There were two Home Rule Leagues.
3. The movement became strong because other mass agitations could not be launched by the congress during the World War.
Select the correct answer using the codes Show Answer


Q191) Which of the following pairs is correctly matched?
1. Jamnalal Bajaj : Satyagraha Ashram
2. Dadabhai Naoroji : Bombay Association
3. Syed Ahmed Khan : MAO College
4. Lala Lajpat Rai : Anusilan Samiti
Select the correct answer using the codes given below Show Answer


Q192) Which one among the following pair is not correctly matched?
1. Lord Wellesley : Subsidiary Alliance
2. William Bentinck : English Education
3. Warren Hastings : Local Self Government
4. Lord Lytton : Vernacular Press Act
Select the correct answer using the codes given below Show Answer


Q193) Consider the following events in the history of Indian Freedom Struggle
1. Champaran Satyagraha
2. Bardoli Satyagraha
3. Ahmedabad Mill Strike
4. Chaura-Chauri Incident
Which one of the following is the correct chronological order, starting from the earliest? Show Answer


Q194) Which of the following statements are correct regarding the Indian Revolutionaries?
1. Khudiram Bose and Prafulla Chaki assassinated Mr Saunders.
2. Attempt on the life of Lt Governor of East Bengal. Mr. Fuller by BK Ghosh and Bhupen Datta.
3. Attempt on the life of Lord Mardings by Ras Behari Bose and Sachin Sanyal.
4. Mr. Kinsgford, the judge of Muzaffarpur was assassinated by Bhagat Singh, Raj Guru as a revenge for the police assault on Lala Lajpat Rai.
Choose the answers from the codes given below Show Answer


Q195) Which one of the pairs is correctly matched?
1. Surendranath Banerjee : Home Rule Movement
2. Lala Hardayal : Organised an army in Europe for India's freedom
3. Raja Radhakanta Deb : Opposed the abolition of the practice of Sati
4. Syed Ahmed Khan : Founder of the Muslim League in 1906
Select the correct answer using the codes given below
Show Answer


Q196) Consider the following statements
1. Ishwar Chandra Vidyasagar founded the Bethune School at Calcutta with the main aim of encouraging education of women.
2. BankiChandra Chattopadhyay was the first graduate of the Calcutta University.
3. Keshab Chandra Sen's campaign against sati led to the enactment of a law to ban Sati by the then Governor-General.
Which of the statements given above is/are correct? Show Answer


Q197) Consider the following statements.
1. In the First Round Table Conference, Dr Ambedkar demanded separate electorates for the depressed classes.
2. In the Poona Act, special provisions of representation of the depressed people in the local bodies and civil services were made.
3. The Indian National Congress did not take part in the Third round Table Conference
Which of the statements given above is/are correct? Show Answer


Q198) Consider the following statements
1. Lord Mountbatten was the viceroy when Simla Conference took place.
2. Indian Navy Revolt, 1946, took place when the India sailors in the Royal Indian Navy at Bombay and Karachi rose against the government.
Which of the statements given above is/are correct? Show Answer


Q199) Consider the following statements.
On the eve of Launch of Quit India Movement, Mahatma Gandhi
1. asked the government servants to resign.
2. asked the soldiers to leave their posts.
3. asked the Princes to the Princely states to accept the sovereignty of their own people.
Which of the statements given above is/are correct? Show Answer


Q200) Consider the following event during India's freedom struggle
1. Chauri-Chaura Outrage
2. Minto-Morley Reforms
3. Dandi March
4. Montague-Chelmsford Reforms
Which one of the following is the correct chronological order of the events above? Show Answer


Q201) Consider the following statements
Some of the main features of the Government of India Act, 1935 were the
1. abolition of diarchy in the Governor's provinces.
2. power of the Governors to veto legislative action and to legislate on their own.
3. abolition of the principle of communal representation.
Which of the statements given above is/are correct?
Show Answer


Q202) An important aspect of Cripps Mission of 1942, was Show Answer


Q203) With reference to the Indian Freedom Struggle, Which one of the following statements is not correct? Show Answer


Q204) With reference to Ryotwari Settlement, consider the following statements
1. The rent was paid directly by the peasants o the government.
2. The government gave pattas to the Ryots.
3. The lands were surveyed and assessed before being taxed.
Which of the statements given above is/are correct? Show Answer


Q205) Mahatma Gandhi said that some of his deepest convictions were reflected in a book titled, Unto This Last and the book transformed his life. What was the message from the book that transformed Mahatma Gandhi? Show Answer


Q206) The members of the Constitution Assembly which drafted the Constitution of India were Show Answer


Q207) Assertion (A) The effect of labour participation in the Indian nationalist upsurge of the early 1930's was weak
Reason (R) The labour leaders considered the ideology of Indian National Congress as Bourgeois and reactionary.
Codes Show Answer


Q208) In collaboration with David Hare and Alexander Duff, who of the following established Hindu College at Calcutta? Show Answer


Q209) who of the following Prime Ministers sent Cripps Mission to India? Show Answer


Q210) The real intention of the British to include the Princely states in the Federal Union proposed by the India Act of 1935, was to Show Answer


Q211) With reference to the Indian freedom struggle, which one of the following statements is not correct? Show Answer


Q212) Who among the following was the President of the all India States Peoples Conference in 1939? Show Answer


Q213) Consider the following statements
1. Arya Samaj was founded in 1835.
2. Lala Lajpat Rai opposed the appeal of Arya Samaj to the authority of Vedas in support of its social reform programme.
3. Under Keshab Chandra Sen, the Brahmo Samaj campaigned for women's education.
4. Vinobha Bhave founded the Sarvodya Samaj to work among refugees.
Which of these statements are correct? Show Answer


Q214) Consider the following statements about the Indian National Congress?
1. Sarojini Naidu as the first woman to be the President of the Congress.
2. CR Das was in prison, when he functioned as President of the Congress.
3. The first Britisher to become the President of the Congress was Alan Octavian Hume.
4. Alfred Webb was the President of the Congress in 1894.
Which of these statements are correct? Show Answer


Q215) Assertion (A) The basic weakness of the early nationalist movement lay in its narrow social base.
Reason (R) It fought for the narrow interest of the social groups which joined it.
Codes. Show Answer


Q216) With which one of the following movements is the slogan Do or Die associated? Show Answer


Q217) Who of the following founded the Ahmedabad Textile Labour Association? Show Answer


Q218) In the Individual Satyagraha, Vinoba Bhave was chosen as the first Satyagrahi. Who was the second? Show Answer


Q219) Assertion (A) Lord Linlithgow described the August Movement of 1942, as the serious revolt after the Sepoy mutiny.
Reason (R) Peasants joined the movement in large number in same place.
Codes Show Answer


Q220) At a time empires in Europe were crumbling before the might of Napoleon, which one of the following Governors-General kept the British flag flying high in India? Show Answer


Q221) 'It made its proposals in May. It still wanted united India. There was to be a Federal Unit composed of British provinces The above quotation is related to Show Answer


Q222) At the time of India's Independence Mahatma Gandhi was Show Answer


Q223) The Indian National Congress agreed in 1947 to the partition of the Country mainly because Show Answer


Q224) On which one of the following dates did Jawaharlal Nehru unfurl the Tricolour National Flag on the bank of the River Ravi as the clock struck midnight? Show Answer


Q225) With reference to the Indian freedom struggle, which one among the following event occurred earliest? Show Answer


Q226) Which one of the following statements is not correct about Dadabhai Naoroji? Show Answer


Q227) Assertion (A) The Congress rejected the Cripps proposal.
Reason (R) The Cripps Mission consisted solely of Whites. Show Answer


Q228) Assertion (A) Partition of Bengal in 1905 brought to an end the Moderates' role in the Indian Freedom Movement.
Reason (R) The Surat Session of Indian Nation congress separated the Extremists from Moderates. Show Answer


Q229) What is the correct sequence of the following events?
1. Tilak's Home Rule League
2. Komagatamaru Incident
3. Mahatma Gandhi's arrival in India
Select the correct answer using the codes given below Show Answer


Q230) The Poona Pact, which was signed between the British Government and Mahatma Gandhi in 1932 provided for Show Answer


Q231) Assertion (A) Khilafat Movement started in India after the Second World War
Reason (R) Gandhiji had been one of the President of the All India Khilafat Conference.
Codes Show Answer


Q232) Who among the following leaders did not believe in the drain theory of Dadabhai Naoroji? Show Answer


Q233) The Anarchical and Revolutionary Crime Act (1919) was popularly known as the Show Answer


Q234) Consider the following landmark in Indian education
1. Hindu College, Calcutta
2. University of Calcutta
3. Adam's Report
4. Wood's Despatch
The correct chronological order of these landmark is Show Answer


Q235) Who among the following was a prominent leader of the Congress Socialist Party? Show Answer


Q236) With reference to the Indian Freedom Struggle, which one of the following is the correct chronological order of the given events? Show Answer


Q237) Which one of the following was provision in the Cabinet Mission Plan? Show Answer


Q238) The offence for which Tilak and others were imprisoned in 1897 was Show Answer


Q239) What is the correct chronological sequence of the following?
1. Wood's Minute Despatch.
2. Macaulay's Minute on education.
3. The Sargeant Education Report.
4. Indian Education (Hunter Commission.
Select the correct answer using the codes given below Show Answer


Q240) Which of the following pairs are correctly matched?
1. Theodore Beck : Mohammadan Anglo-Oriental College, Aligarh
2. Ilbert Bill : Ripon
3. Pherozeshah Mehta : Indian national Congress
4. Badruddin Tyabji : Muslims League
Select the correct answer using the codes given below Show Answer


Q241) Who were the main architects of the All Indian National Conference? Show Answer


Q242) Who among the following repealed the Vernacular Press Act? Show Answer


Q243) Consider the following statements
1. Lord Mount Batten was sent to India in place of Lord Wavell as the Governor-General of India in March 1947.
2. In July 1947, British Government passed the Act, containing the main provisions of the Mount Batten Plan This is known as Indian Independence Act.
Which of the statements given above is/are correct? Show Answer


Q244) Consider the following statements
1. The Constituent assembly started functioning in December 1946.
2. Jawaharlal Nehru was the Chairman of the Constituent Assembly.
3. The Constitution Assembly adopted the Constitution of India on 26th November, 1949.
Select the correct answer using the codes given below Show Answer


Q245) Consider the following statements
1. Subhash Chandra Bose resigned from the office of the President of INC in the year 1939.
2. Subhash Chandra Bose left the Congress to form the Indian Independence League in South East Asia.
Which of the statements given above is/are correct? Show Answer


Q246) Consider the following statements
1. The British crown has assumed direct rule in India before the Sepoy mutiny.
2. Lord Canning was made the first Viceroy and Governor-General after the end of rule of the East India Company.
Select the correct answer using the codes given below Show Answer


Q247) Which of the following is not correctly matched? Show Answer


Q248) Which portfolio was held by Dr Rajendra Prasad in the Interim Government formed in the year 1946? Show Answer


Q249) At which one of the following Congress sessions did Dadabhai Naoroji announce the Swaraj was the goal of India's political efforts? Show Answer


Q250) Who among the following was not associated with the Indigo Rebellion? Show Answer


Q251) Who was the President of INC at the time of partition of India? Show Answer


Q252) Consider the following statement and identify the author of the statement using the codes given below
"I shall work for an India in which the poorest shall feel that it is their country, in whose making they have an effective volice, an India in which there shall be no high class and low class of people, an India in which all communities shall live in perfect harmony" Show Answer


Q253) Which of the following is the correct chronological order of the given events?
1. Champaran Satyagraha
2. Moplah Rebellion
3. Jallianwala Bagh massacre
Select the correct answer using the codes given below Show Answer


Q254) Which of the following is the correct Chronological order of the given events?
1. Nasik Conspiracy
2. Kakori Conspiracy
3. Second Lahore Conspiracy
The Correct Answer is Show Answer


Q255) Which of the following statements is not correct? Show Answer


Q256) Consider the following statements
1. Burma and India separated under the Government of India Act, 1919.
2. The Government of India Act, 1935 provided for the abolition of the Indian Council.
3. The Government of India Act, 1935 granted limited Franchise to Indian women.
Which of the statements given above is/are correct? Show Answer


Q257) When Cripp's Mission came to India, who among the following was the Viceroy of India? Show Answer


Q258) Which of the following founded a national Society, a National Paper, a National School and National Gymnasium and made the word 'National popular in the later half of the 19th Century? Show Answer


Q259) Which one of the following leaders of the Congress was totally in favour of the Cabinet Mission Plan? Show Answer


Q260) Which one of the following Indian leaders was dismissed by the British from the Indian Civil Service? Show Answer


Q261) The term imperial preference was applied to the Show Answer


Q262) The historical importance of the second session of the INC held in Calcutta in 1886 was that Show Answer


Q263) The Congress policy of pray and petition ultimately came to an end under the guidance of Show Answer


Q264) "In this instance we could not play off the Mohammadans against the Hindus". To which one of the following events did this remark of Aitchison relate?
Show Answer


Q265) Which one of the following is not a feature of the Government of Indian Act of 1935? Show Answer


Q266) The Indian National Army (INA) came into existence in 1943 in Show Answer


Q267) Who among the following suggested the winding up of the Indian National Congress after India attained independence? Show Answer


Q268) Which of the following statements about Annie Besant are correct?
1. She founded the Central Hindu College at Benaras.
2. She organised the Home Rule League.
3. She introduced the Theosophical movement in India.
Select the correct answer using the codes given below Show Answer


Q269) Consider the following statements about Jawaharlal Nehru
1. He was the President of the Congress Party in 1947.
2. He presided over the Constitution Assembly.
3. He formed the first Congress ministry in United.
Which of the above statements are correct? Show Answer


Q270) Which one of the pairs is not correctly matched? Show Answer


Q271) BR Ambedkar was elected to thee Constituent Assembly from Show Answer


Q272) Consider the following pairs
1. Non-Cooperation Movement was called off : Chauri Chaura
2. Ilbert Bill : Lord Hastings
3. August Offer : Lord Rippon
Which of the pair/s given above is/are correct? Show Answer


Q273) Consider the following pairs
1. Founder of East India Association : Dadabhai Naoroji
2. Vande Matram : Bipin Chandra Pal
3. Lokhitwadi : Gopal Hari Deshmukh
Select the correct answer using the codes given below
Show Answer


Q274) which of the following is not matched correctly? Show Answer


Q275) Consider the following
1. Rowlatt Act Movement
2. Khera Movement
3. Champaran
4. Ahmedabad Strike
What is the correct chronological order of the above mentioned Gandhian Movements? Show Answer


Q276) Raja Ram Mohan Roy was responsible for which one of the following reforms?
1. Abolition of Sati
2. Widow Remarriage
3. To Purity Hinduism
4. To Preach Monotheism
Select the correct answer using the codes given below?
Show Answer


Q277) With reference to 'Kamraj Plan' which one of the following sentences is not correct? Show Answer


Q278) Consider the following statements
1. William Bentinck introduced English as the medium of instruction in India.
2. William Bentinck wanted to promote Western culture in India.
Which of the statements given above is/are correct? Show Answer


Q279) With reference to the period of colonial rule of India, consider the following statements
1. Raja Ram Mohan Roy established the Calcutta Unitarian Committee.
2. Social Service League was established by MG Ranande.
3. Decon Education Society was founded by G K Gokhale.
4. Dr Annie Beasant started the Central Hindu School.
Which of the statements given above is/are correct?
Show Answer


Q280) Consider the following statements
1. The Government of India Act, passed in 1935, gave dominion status to India.
2. The INC under Motilal Nehru claimed Swarajya and drafted a Constitution for free India.
Which of the statements given above is/are correct? Show Answer


Q281) During the Indian Freedom Struggle, who among the following was the first to use to word Swarajya for the demand of full Indian control over all legislation and finances, on the model of the self-governing colonies of Canada and Australia? Show Answer


Q282) With reference to the Indian Freedom Struggle, consider the following statements
1. The Cabinet Mission plan stated that the Muslim League's demand for Pakistan could not be conceded.
2. The Cabinet Mission plan proposed a Federal Union embracing both the British India and the Princely States.
Which of the statements given above is/are correct? Show Answer


Q283) In the context of the Indian Freedom Struggle the famous 3rd June Plan is also known as which one of the following? Show Answer


Q284) Consider the following statements in regard to the Kheda Satyagraha Movement in India
1. The term Satyagraha was used for the first time in India during this movement.
2. During the movement, Vallabhbhai Patel was honoured with the title of Sardar.
Which of the statements given above is/are correct? Show Answer


Q285) In the beginning, the political activities of the Muslim League were directed against Show Answer


Q286) What was the basic weakness of the Muslim League, which came to be increasingly recognised by the patriotic Muslims? Show Answer


Q287) Consider the following statements
1. The name Indian National Congress was given on the suggestion of GK Gokhale.
2. The Brahmins, among the castes, were comparatively large in number in INC.
3. The foundations of British Committee of the Congress by Dada Bhai Naoroji, AO Hume and William Wedderburn was done to influence British public opinion at London.
Select the correct answer using the codes given below
Show Answer


Q288) Consider the following statements
1. Mahatma Gandhi attended the First Round Table Conference as the sole representative of Indian National Congress.
2. Sir Tej Bahadur Sapru and Sir Mohammad Shafi attended the First Round Table Conference as representatives of Indian National Congress.
Which of the statements given above is/are correct? Show Answer


Q289) Which one of the following was the purpose for which a highlevel committee chaired by Balwant Rai Mehta was constituted during the time of Jwaharlal Nehru as Prime Minister? Show Answer


Q290) Arrange the following chronologically
1. Land Holders Society at Calcutta.
2. Bengal British India Society.
3. British India Association.
4. Madras Native Association.
5. East India Association in London.
Codes Show Answer


Q291) The people of India agitated against the arrival of Simon Commission because Show Answer


Q292) Annie Besant was
1. responsible for starting the Home Rule movement.
2. the founder of the Theosophical Society.
3. once the president of the Indian National Congress.
Select the correct answer using the codes given below Show Answer


Q293) The demand for the Tebhaya Peasant Movement in Bengal was for Show Answer


Q294) Consider the following statements
1. When Shyama Prasad Mukherji resigned from the Nehru Ministry, he founded the Kisan Mazdoor Praja Party.
2. When JB Kriplani left the Congress Party, he founded the Jana Sangh.
Which of the statements given above is/are correct? Show Answer


Q295) Which of the following statements is/are correct about Khera Movement?
1. This movement failed completely due to the repressive measure of the government.
2. The Khera Peasants launched a no-revenue campaign under the leadership of Gandhi and Vallabh Bhai Patel.
3. It was caused by the government refusal to exempt the peasants of Khera from Revenue payment despite the failure due to drought.
Select the correct answer using the codes given below
Show Answer


Q296) Consider the following statements relating to Raja Ram Mohan Roy
1. He play a pioneering role in the Religious and Social Reform Movement of 19th century Bengal.
2. He know that the spread of westers education was necessary to develop a national and scientific attitude in the Indian Society.
3. He founded Atmiya Sabha in Calcutta in 1815 to propagate monotheism and reforms in the Hindu society.
Which of the statements given above is/are correct? Show Answer


Q297) Consider the following statements
1. Gandhiji finally accepted the Partition of India because it seemed in the circumstances.
2. In the Interim Government formed in the year 1946, Jagjivan Ram held the portfolio of defence.
Which of the statements given above is/are correct? Show Answer


Q298) Consider the following statements
1. The Munddiman Committee was appointed in 1942 to report on the functioning of the reforms of 1919.
2. In 1924, the Swarajists accepted seats on the steel protection committee.
3. In 1925, Jawahar Lal Nehru accepted membership of the Skeen Committee which was appointed to find out the possibility of rapid Indianisation of army.
4.The Swarajist Party was formed by CR Das and Pt Motilala Nehru.
Choose the answer from the codes given below. Show Answer


Q299) Quit India Movement was launched in response to Show Answer


Q300) Assertion (A) Gandhi undertook a fast unto death in protest against the Communal Award.
Reason (R) The Communal Award proposed to introduce separate electorates for the untouchables.
Codes Show Answer


Q301) Which of the following statements about factory commissions are correct?
1. On the basis of the recommendations of the Second Commission the Second Factory Act was passed in 1890.
2. On the basis of the recommendation of the First Commission, the First Factory Act was passed in 1887.
3. The First Factory Commission was appointed in 1875.
4. The Second Factory commission was appointed in 1886.
Choose the right answer from the codes given below Show Answer


Q302) Assertion (A) Mrs Annie Besant organised the Home Rule Movement against the British.
Reason (R) She wanted to organise all section of people on the basis of a single slogan.
Codes Show Answer


Q303) Assertion (A) The revolt of 1857 was the first struggle of the Indians to throw off British Yoke.
Reason (R) The soldiers who revolted were inspired by nationalistic feelings.
Codes Show Answer


Q304) With reference to the Government of India Act, 1935, consider the following statements
1. The Legislative Assembly having not more than 125 representatives of British India.
2. The King represented by the Governor-General.
3. The Council of State having ISO representatives of British India and another ISO Representatives of the Indian States.
Which of the statements given above is/are correct? Show Answer


Q305) Muslim Faqirs form their headquarters in the Nepal Terai, wandered about the country levying contributions and defying the authority of the British Government. Who of the following was their founder leader? Show Answer


Q306) Who presided over the first session of All India Kisan Sabha? Show Answer


Q307) Which British commander was defeated by the Santhals in 1855? Show Answer


Q308) Assertions (A) Many tribal uprisings were caused by the restriction imposed on the use and access to forest.
Reason (R) The Company's Governments forest policies were directed towards establishing greater control over forest lands.
Codes Show Answer


Q309) The first President of All India Trade Union Congress (AITUC) was Show Answer


Q310) Kuka Movement was organised by Show Answer


Q311) In which of the following years was the All India Trade Union Congress (AITUC) formed in Bombay? Show Answer


Q312) The first All India Trade Union Congress (AITUC) was founded in 1920 by Show Answer


Q313) During Indian Freedom struggle, a general strike jointly called by the Girni Kamgar Union and the railway workers in March, 1929 in Bombay. This was actually in protest against
Show Answer


Q314) which one of the following observations is not true in respect to the Indigo Revolt of 1859-60 in India? Show Answer


Q315) Consider the following statements
1. The All India Union Congress was formed in 1920.
2. Lokmanya Tilak, played an important role in the formation of the AITUC.
3. Lala Lajpat Rai was appointed as its first President.
Which of the statements given above is/are correct?
Show Answer


Q316) Assertion (A) The Akhil Bhartiya Kisan Sabha succeeded to organise the Indian peasants to struggle against the British rule on a national scale.
Reason (R) The abolition of the self-dependent economy of the Indian villages under the British rule was a progressive step towards the economic unification of rural India.
Codes Show Answer


Q317) Who among the the following was not a leader of the All India Kisan Sabha? Show Answer


Q318) Which of the following pairs is not correctly matched ? Show Answer


Q319) The Gadhkari Uprisings (1844-45) took place in Show Answer


Q320) The main cause of Sanyasi Rebellion was Show Answer


Q321) The rebellion started in 1816 and continued till 1832 was Show Answer


Q322) Who among the following was the President of the first session of All India Trade Union Congress? Show Answer


Q323) Under whom was Kanpur Labour Enquiry Committee constituted Show Answer


Q324) Which of the following statements about All India Trade Union Congress (AITUC) is not true?
1. AITUC was affiliated to Pan-Pacific secretaries and to the third international.
2. The reformers group of AITUC wanted affiliation with the International Federation of Trade.
3. The revolutionary group of AITUC wanted affiliation with Red Labour Union. Show Answer


Q325) What was the Main objective of Lord Wellesley in concluding a subsidiary treaty (1798), with the Nizam? Show Answer


Q326) Consider the following statements
1. 1757-1813 period is known as period of mercantilism.
2. 1860-1947 period is called the period of Financial Capitalism.
Which of the statement(s) given above is/are correct? Show Answer


Q327) The true significance of the Treaty of Amritsar was Show Answer


Q328) Consider the following Princely States of the British rule in India.
1. Jhansi
2. Sambalpur
3. Satara
The correct chronological order in which they were annexed by the British is
Show Answer


Q329) The ruler of which one of the following States was removed from power by the British on the pretext of misgovernance? Show Answer


Q330) Consider the following statements about the Deccan Riots(1875).
1. It was a peasant movement of the poor tribals or landless peasants.
2. It was led by the relatively better-off sections of the peasantary, who were called Kunvi peasants in Maharashtra.
Which of the statement(s) given above is/are correct? Show Answer


Q331) Through whose initiative was the Dhaka Anushilan Samiti formed in October, 1906? Show Answer


Q332) The Gadkari uprising of 1844-45 was directed against the Show Answer


Q333) A popular movement of the 19th Century, which was much better planned, organised and knit than Revolt of 1857 was Show Answer


Q334) The Ahrar Movement Show Answer


Q335) Which among the following was the First Trade Union Organisation of India in modern times? Show Answer


Q336) The Federation of Indian Chamber of Commerce and Industry (FICCI) was founded in 1927 on the advice of MK Gandhi by Show Answer


Q337) Which of the following is/are the provision(s) of the Regulating Act, 1773?
1. The act remodeled the Constitution of the company both in England and in India.
2. It was provided that the Court of Directors, Hitherto elected every year, was henceforth to be elected for 1 year.
3. The number of directors was fixed at 24 and one-third of them retiring every four years.
Select the correct answer using the codes given below
Show Answer


Q338) Which of the following revolted first against British? Show Answer


Q339) The state of Jhansi was made a part of the British Empire in India through Show Answer


Q340) With reference to the colonial rule of India, which one of the following was not the feature of Subsidiary Alliance System? Show Answer


Q341) Consider the following statements
1. The treaty of Lahore was concluded between Sikhs and English Company during the tenure of Lord Ellenborough.
2. Lord Dalhousie made the annexation of Punjab to the British dominion.
Which of the statement(s) given above is/are correct? Show Answer


Q342) With reference to the Act of 1858 and the secretary of State for India, consider the following statements
1. In laying the policy respecting the Indian States and in making war or peace, it was obligatory for the Secretary of State for India to obtain the concurrence of his Council of India.
2. Before any expenditure out of Indian revenues was incurred or loans could be raised on their security, the conference of the Council of India was not necessary for the Secretary of State of India.
Which of the statement(s) given above is/are correct?
Show Answer


Q343) The successor of Maharaja Ranjit Singh was Show Answer


Q344) Consider the following statements
1. In the Wahabi uprising the leader Chirag Ali was supported by two leaders namely Bhawani Pathak and Devi Chaududhrani.
2. The real founder of Faraizi Movement was Haji Shariat Ullah of Faridpur.
3. Maria was a practice of human sacrifice followed by the Khonds tribe of Orissa.
Select the correct answer using the codes given below Show Answer


Q345) Read the following passage
"He was the most liberal and enlightened Governor-General of India, regarded as" the father of Modern Western Education in India, he abolished sati and passed the Charter Act of 1833, which provided that no Indian subject of company was to be debrred form holding an office on account of his religion, place of birth descent and colour.
The Passage above is referring to Show Answer


Q346) which Governor-General said the following statements "An insult offered to the British flag at the mouth of the Ganges should be resented as promptly and fully as an insult offered at the mouth of the Thames." Show Answer


Q347) Consider the following statements
1. Sambhaji granted permission to the French to fortify Pondicherry.
2. On recommendation of Dost Ali, Mughal emperor Muhammad Shah issued Firman to French to mint and issue gold and silver coins.
Which of the statement(s) given above is/are correct? Show Answer


Q348) With reference of Ryotwari Settlement consider the following statements
1. The rent was paid directly by the peasants the government.
2. The government gave Pattas to the Ryots.
3. The land were surveyed and assessed before being taxed.
Which of the statement(s) given above is/are correct? Show Answer


Q349) Consider the following statements
1. British intervention in Afghan gave birth to three Afghan wars.
2. Lord Lawrence initiated the policy of masterly inactivity that refers to our intervention but being watchful.
3. Lord Ellenborough was responsible for the plan of conquest of Sindh.
Which of the statement(s) given above is/are correct? Show Answer


Q350) With reference to Permanent Settlement, consider the following statements
1. The Zamindars were recognised owners of land and a ten year's settlement was made with them in 1790.
2. The decennial settlement was declared permanent in 1793 and the Zamindars and their legitimate successors were allowed to hold their estates at the very assessed rate forever.
3. The state demands was fixed at 11% of the rental, leaving 81% with the Zamindars as their share.
Which of the statement(s) given above is/are correct? Show Answer


Q351) The Dastaks issued by the Nizam to the company involved conflict. Why it is so? Show Answer


Q352) which of the following statements is correct? Show Answer


Q353) Consider the following statements
1. Nizam of Hyderabad was the first Indian native ruler to accept the system of subsidiary alliance.
2. The Gurkha regiment was raised by Lord Cornwallis.
3. Lord canning withdrew Doctrine of Lapse introduced by Lord Dalhousie.
Which of the statement(s) given above is/are correct? Show Answer


Q354) Consider the following statements about Lord warren Hastings.
1. First Rohilla War.
2. Strained relationship with Maharaja of Benaras.
3. Foundation of Asiatic Society of Bengal.
Which of the statement(s) given above is/are correct? Show Answer


Q355) Consider the following statements
1. The Sikh leader Banda Bahadur was captured and killed during the reign of Emperor Farrukhsiyar.
2. The Marathas fought with Ahmed Shah Abdali at Panipat during the reign of Emperor Shah Alam II.
Which of the statement(s) given above is/are correct? Show Answer


Q356) Assertion (A) The state of Hyderabad was founded by Nizam-ul-Mulk, a wazir of Mughal Emperor.
Reason (R) The Mughal Emperor frustrated all the attempts of Nizam-ul-Mulk at reforming the administration of the empire. Show Answer


Q357) In whose reign, Guru Nanak Dev established Sikhism? Show Answer


Q358) Who of the following compiled Adigranth of Guru Granth Saheb? Show Answer


Q359) Which of the following was capital of Raja Ranjit Singh of Punjab? Show Answer


Q360) The book Zij Mohammad Shahi related to knowledge of Astrology produced in 1733 is written by Show Answer


Q361) Who converted Sikhs into a martial race? Show Answer


Q362) Who founded independent state of Bengal? Show Answer


Q363) Who founded Sikhism? Show Answer


Q364) Which Sikh Guru assumed the title 'Sachcha Badshah'? Show Answer


Q365) Who among the following was not involved in conspiracy against Siraj-ud-Daullah? Show Answer


Q366) The Gurumukhi script was introduced by Show Answer


Q367) Who was the last independent Nawab of Bengal? Show Answer


Q368) Assertion (A) Shah Alam II spent the initial years as an Emperor far away from his capital
Reason (R) There was always a lurking danger of foreign invasion from the North-West frontier.
Codes Show Answer


Q369) Consider the following statements about Sikhs Gurus
1. Banda Bhadur was appointed as a military leader of the Sikhs by Guru Teg Bahadur.
2. Guru Arjan Dev became the Sikh Guru after Guru Ramdas.
3. Guru Angad/Anga Dev gave the Sikhs their own scipt Gurumukhi.
Which of the statement(s) given above is/are correct? Show Answer


Q370) The recognised Bombay Textile Mills Labour Union registered in 1926 was led by Show Answer


Q371) Which of the following was not amongst the provision of Pitt's India Act? Show Answer


Q372) consider the following statements
1. In the year 1919, MA Jinnah and MM Malaviya resigned their membership of central Legislative Council.
2. In the year, the Government of India passed the Rowlatt Act, which was opposed by all the Indian members of the Legislative Council
which of the above statement is /are correct? Show Answer


Q373) Consider the following statements
1) The National Anthem Jana-gana-mana is composed originally in Sanskrit by Rabindranath Tagore.
2) The National Anthem was sung for the first time in the Kolkata Session of the Indian National Congress in 1911.
Which of the statements given above is/are correct ? Show Answer


Q374) Who did not participate in the Revolt of 1857 ? Show Answer


Q375) The main weakness of Revolt of 1857 was lack of— Show Answer


Q376) Gandhiji first experiment with Satyagraha came up in— Show Answer


Q377) During British Era, the Duke Memorandum became the basis of which among the following? Show Answer


Q378) Which book was written by Dadabhai Naoroji? Show Answer


Q379) On November 1, 1858, Queen Victoria Proclamation was read out at Allahabad by— Show Answer


Q380) Which one of the following was the first to impose censorship of the press ? Show Answer


Q381) Who was the founder of Prarthna Samaj ? Show Answer


Q382) Who founded ‘Anushilan Samiti’ ? Show Answer


Q383) The American publicist who was with Mahatma Gandhi during his ‘Quit India’ movement was— Show Answer


Q384) Who was the first President of All India Trade Union Congress— Show Answer


Q385) With reference to Lord Cornwallis's permanent settlement, which one of the following statements is not correct ? Show Answer


Q386) Of the following who was not a signatory to the historic Poona Pact of 1932 ? Show Answer


Q387) Which one of the following may be regarded as the first labour association in India ? Show Answer


Q388) Who accused Indian National Congress of practising ‘politics’ of prayer, petition and protest— Show Answer


Q389) Who converted Sikhs into a martial race ? Show Answer


Q390) Aim of the Swarajya Party was to— Show Answer


Q391) The Partition of Bengal made by Lord Curzon in 1905 lasted until Show Answer


Q392) The 1929 Session of Indian National Congress is of significance in the history of the Freedom Movement because the
Show Answer


Q393) The people who lived in the subcontinent as early as two million years ago are described as- Show Answer


Q394) Which of the following statement is not correct regarding Rowlatt Act? Show Answer


Q395) The Governor-General associated with the prohibition of Sati was— Show Answer


Q396) An anti British outfit 'Abhinava Bharat' was founded by— Show Answer


Q397) Which of the following represented the Indian Christians in the First Round Table Conference held at London ? Show Answer


Q398) Which one of the following is not a feature of the Government of India Act of 1935? Show Answer


Q399) Consider the following statements-
1. On 8 August 1942, the Quit India resolution was passed at the Bombay session of the All India Congress Committee (AICC).
2. The draft proposed that if the British did not accede to the demands, a massive Civil Disobedience would be launched. However, it was an extremely controversial decision.
3. The British, already alarmed by the advance of the Japanese army to the India–Burma border, responded the next day by imprisoning Gandhi at the Aga Khan Palace in Pune.
Select the correct answer using the codes given below. Show Answer


Q400) Which one of the following statements is not correct?
1. The Regulating Act of 1773 was enacted to primarily weed out corruption in the
British East India Company.
2. The Act had laid down specific and drastic rules for officials and the Company itself.
3. The Amending Act of 1781 had come as a sequential outcome to the previous Regulating Act which for the first time made the provision for the post of Governor-General in India.
4. The Regulating Act was a short document.
Which of the above pairs is/are correctly matched? Show Answer


Q401) Consider the following statements
1. The Khilji dynasty were the second Muslim dynasty to rule the Delhi Sultanate.
2. The slave rulers laid a firm foundation to the Delhi Sultanate.
3. Naturally Muslims from territories bordering to western northern India migrated to join other Muslim settlers.
Select the correct answer using the codes given below. Show Answer


Q402) Consider the following statements-
1. In the middle of the 18th century, the French and the British East India company initiated a protracted struggle for military control of South India.
2. The period was marked by shifting alliances between the two East India companies and the local powers, mercenary armies employed by all sides, and general anarchy in South India.
3. Cities and forts changed hands many times, and soldiers were primarily remunerated through loot.
4. The four Anglo-Mysore Wars and the three Anglo-Maratha Wars saw Mysore, the Marathas and Hyderabad aligning themselves in turns with either the British or the French.
Select the correct answer using the codes given below. Show Answer


Q403) Consider the following statements-
1. South India during the British colonial rule was divided into the Madras Presidency and Hyderabad, Mysore, Thiruvithamcoore (also known as Travancore), Cochin, Vizianagaram and a number of other minor princely states.
2. The Madras Presidency was ruled directly by the British, while the rulers of the princely states enjoyed considerable internal autonomy.
3. British Residents were stationed in the capitals of the important states to supervise and report on the activities of the rulers. British troops were stationed in cantonments near the capitals to curb the potential of rebellion.
Select the correct answer using the codes given below. Show Answer


Q404) Consider the following statements-
1. On August 15, 1947, the former British India achieved independence as the new dominions of India and Pakistan.
2. The rulers of India's princely states acceded to the government of India between 1947 and 1950, and South India was organized into a number of new states.
3. Most of South India was included in Madras state, which included the territory of the former Madras Presidency together with the princely states of Banganapalle, Pudukkottai, and Sandur.
Select the correct answer using the codes given below. Show Answer


Q405) Consider the following statements-
1. In 1953, the Jawaharlal Nehru government yielded to intense pressure from the northern Telugu-speaking districts of Madras State, and allowed them to vote to create India's first linguistic state.
2. Andhra State was created on October 1, 1953 from the northern districts of Madras State, with its capital in Kurnool. Increasing demands for reorganisation of the patchwork of India's states resulted in the formation of a national States Reorganisation Commission.
3. Based on the commission's recommendations, the Parliament of India enacted the States Reorganisation Act of 1956, which reorganized the boundaries of India's states along linguistic lines.
4. Andhra State was renamed Andhra Pradesh, and enlarged by the addition of Telugu-speaking region of Telingana, formerly part of Hyderabad State.
Select the correct answer using the codes given below. Show Answer


Q406) The Quit India Movement was launched in 1942 In this context consider the following statements -
1. The Quit India Movement was a civil disobedience movement launched in response to Mohanda
Gandhi's call for 'Satyagraha'
2. The call for determined, but passive resistance appears in Mohandas Gandhi's call to Do or Die, issued on
August in Mumbai..
3. Almost the entire INC leadership, and not just at the national level, was imprisoned without trial
4. The Quit India campaign was effective and successful
Select the correct answer using the codes given below. Show Answer


Q407) Consider the following statements
1. The Indian independence movement was a mass-based movement that encompassed various sections of society.
2. It also underwent a process of constant ideological evolution.
3. Although the basic ideology of the movement was anti-colonial, it was supported by a vision of independent capitalist economic development coupled with a secular, democratic, republican, and civil-libertarian political structure.
4. After the 1930s, the movement took on a strong socialist orientation, due to the increasing influence of left-wing elements in the INC as well as the rise and growth of the Communist Party of India.
Select the answer using the codes given below- Show Answer


Q408) Consider the following statements:
1. The Indian rebellion of 1857 was a large-scale rebellion in northern and central India against the British East India Company's rule.
2. The conditions of service in the company's army and cantonments increasingly came into conflict with the religious beliefs and prejudices of the sepoys.
3. The predominance of members from the upper castes in the army, perceived loss of caste due to overseas travel, and rumours of secret designs of the government to convert them to Christianity led to deep discontentment among the sepoys.
4. The sepoys were also disillusioned by their low salaries and the racial discrimination practised by British officers in matters of promotion and privileges.
5. The indifference of the British towards leading native Indian rulers such as the Mughals and ex-Peshwas and the annexation of Oudh were political factors triggering dissent amongst Indians.
Select the correct answer using the codes given below. Show Answer


Q409) Consider the following statements:
1. Revolts broke out in other parts of Oudh and the North-Western Provinces as well, where civil rebellion followed the mutinies, leading to popular uprisings.
2. The British fought the main army of the rebels near Delhi, and after prolonged fighting and a siege, defeated them and retook the city on 20 September 1857.
3. The last significant battle was fought in Gwalior on 17 June 1858, during which Rani Lakshmibai was killed.
4. Sporadic fighting and guerrilla warfare, led by Tatya Tope, continued until spring 1859, but most of the rebels were eventually subdued.
Select the answer using the codes given below- Show Answer


Q410) Consider the following statements-
1. Of all the provincial styles that developed in architecture in the mid-14th century, the Gujarat style is the most elegant.
2. The Gujarat rulers evolved a synthesis of Islamic and Jain styles.
3. The characteristic features of this style include extensive use of pillar-and¬lintel system, device (a clerestory or wall with series of windows) to admit light into the central pan of the hall¬type mosque, semi-circular engrailed arch on two pillars, systematic use of graceful minarets and heavily sculpted buttresses, rich and delicate jali in arches, windows, etc. and frequent use of balconied windows.
4. Some fine examples of this style are the Rani Rupmati (c. 1460) mosque and the tomb and mosque of Rani Sabrai or Rani Sarai (1514).
Select the correct answer using the codes given below. Show Answer


Q411) Consider the following statements:
1. Under the Government of India Act 1858, the Company was deprived of its involvement in ruling India, with its territory being transferred to the direct authority of the British government.
2. At the apex of the new system was a Cabinet minister, the Secretary of State for India, who was to be formally advised by a statutory council; the Governor-General of India (Viceroy) was made responsible to him, while he in turn was responsible to the British Parliament for British rule.
Which of the statements given above is/are correct? Show Answer


Q412) Consider the following statements:
1. Inspired by a suggestion made by A.O. Hume, a retired British civil servant, seventy-three Indian delegates met in Bombay in 1885 and founded the Indian National Congress.
2. They were mostly members of the upwardly mobile and successful western-educated provincial elites, engaged in professions such as law, teaching and journalism.
3. At its inception, the Congress had no well-defined ideology and commanded few of the resources essential to a political organisation.
4. Instead, it functioned more as a debating society that met annually to express its loyalty to the British Raj and passed numerous resolutions on less controversial issues such as civil rights or opportunities in government (especially in the civil service).
Select the answer using the codes given below- Show Answer


Q413) Consider the following statements:
1. Bal Gangadhar Tilak was the first Indian nationalist to embrace Swaraj as the destiny of the nation
2. Tilak deeply opposed the then British education system that ignored and defamed India's culture, history and values.
3. He resented the denial of freedom of expression for nationalists, and the lack of any voice or role for ordinary Indians in the affairs of their nation.
4. His popular sentence "Swaraj is my birthright, and I shall have it" became the source of inspiration for Indians.
Select the answer using the codes given below- Show Answer


Q414) Consider the following statements:
1. In July 1905, Lord Curzon, the Viceroy and Governor-General (1899–1905), ordered the partition of the province of Bengal supposedly for improvements in administrative efficiency in the huge and populous region.
2. It also had justifications due to increasing conflicts between Muslims and dominant Hindu regimes in Bengal.
3. the Indians viewed the partition as an attempt by the British to disrupt the growing national movement in Bengal and divide the Hindus and Muslims of the region
4. Brahmabhandav Upadhyay, a Hindu newspaper editor who helped Tagore establish his school at Shantiniketan, was imprisoned and the first martyr to die in British custody in the 20th century struggle for independence.
Select the answer using the codes given below- Show Answer


Q415) Consider the following statements:
1. The All India Muslim League was founded by the All India Muhammadan Educational Conference at Dhaka (now Bangladesh), in 1906, in the context of the circumstances that were generated over the partition of Bengal in 1905.
2. Being a political party to secure the interests of the Muslim diaspora in British India, the Muslim League played a decisive role during the 1940s in the Indian independence movement and developed into the driving force behind the creation of Pakistan in the Indian subcontinent.
3. But when Muslim league passed Pakistan resolution based on Two Nation theory of Jinnah, Nationalist leaders like Maulana Azad and others stood against it.
4. All-India Jamhur Muslim League was formed parellal to Muslim League with Raja of Mahmoodabad (a close associate of Jinnah) as its president and Dr.Maghfoor Ahmad Ajazi its general secretary.
Select the answer using the codes given below- Show Answer


Q416) Consider the following statements:
1. The First World War began with an unprecedented outpouring of love and goodwill towards the United Kingdom from within the mainstream political leadership, contrary to initial British fears of an Indian revolt.
2. India contributed massively to the British war effort by providing men and resources.
3. 1.3 million Indian soldiers and labourers served in Europe, Africa and the Middle East, while both the Indian government and the princes sent large supplies of food, money and ammunition.
4. Bengal and Punjab remained hotbeds of anti colonial activities.
5. Nationalism in Bengal, increasingly closely linked with the unrests in Punjab, was significant enough to nearly paralyse the regional administration.
Select the correct answer using the codes given below. Show Answer


Q417) Consider the following statements:
1. The British themselves adopted a "carrot and stick" approach in recognition of India's support during the war and in response to renewed nationalist demands.
2. In August 1917, Edwin Montagu, the secretary of state for India, made the historic announcement in Parliament that the British policy for India was "increasing association of Indians in every branch of the administration and the gradual development of self-governing institutions with a view to the progressive realization of responsible government in India as an integral part of the British Empire."
3. The means of achieving the proposed measure were later enshrined in the Government of India Act 1919, which introduced the principle of a dual mode of administration, or diarchy, in which both elected Indian legislators and appointed British officials shared power.
Which of the statements given above is/are correct? Show Answer


Q418) Consider the following statements:
1. Gandhi's ideas and strategies of non-violent civil disobedience initially appeared impractical to some Indians and Congressmen.
2. In Gandhi's own words, "civil disobedience is civil breach of unmoral statutory enactments."
3. It had to be carried out non-violently by withdrawing cooperation with the corrupt state.
4. Gandhi's ability to inspire millions of common people became clear when he used satyagraha during the anti-Rowlatt Act protests in Punjab.
5. Gandhi had great respect for Lokmanya Tilak. His programmes were all inspired by Tilak's "Chatusutri" programme.
Select the correct answer using the codes given below. Show Answer


Q419) Consider the following statements:
1. The positive impact of reform was seriously undermined in 1919 by the Rowlatt Act, named after the recommendations made the previous year to the Imperial Legislative Council by the Rowlatt Commission.
2. The Rowlatt Act vested the Viceroy's government with extraordinary powers to quell sedition by silencing the press, detaining the political activists without trial, and arresting any individuals suspected of sedition or treason without a warrant. In protest, a nationwide cessation of work (hartal) was called, marking the beginning of widespread, although not nationwide, popular discontent.
3. The agitation unleashed by the acts led to British attacks on demonstrators, culminating on 13 April 1919, in the Jallianwala Bagh massacre (also known as the Amritsar Massacre) in Amritsar, Punjab.
4. The British military commander, Brigadier-General Reginald Dyer, blocked the main, and only entrance-cum-exit, and ordered his soldiers to fire into an unarmed and unsuspecting crowd of some 15,000 men, women and children.
Select the correct answer using the codes given below. Show Answer


Q420) Consider the following statements:
1. The independence movement as late as 1918 was an elitist movement far removed from the masses of India, focusing essentially on a unified commerce-oriented territory and hardly a call for a united nation. Gandhi changed all that and made it a mass movement.
2. At the Calcutta session of the Congress in September 1920, Gandhi convinced other leaders of the need to start a non-cooperation movement in support of Khilafat as well as for swaraj (self rule).
3. The first satyagraha movement urged the use of khadi and Indian material as alternatives to those shipped from Britain urging people to boycott British educational institutions and law courts; resign from government employment; refuse to pay taxes; and forsake British titles and honours.
Which of the statements given above is/are correct? Show Answer


Q421) About which act do the above mentioned statements refer to- Show Answer


Q422) Consider the following statements:
1. Following the rejection of the recommendations of the Simon Commission by Indians, an all-party conference was held at Bombay in May 1928.
2. This was meant to instill a sense of resistance among people.
3. The conference appointed a drafting committee under Motilal Nehru to draw up a constitution for India.
4. The Calcutta session of the Indian National Congress asked the British government to accord dominion status to India by December 1929, or a countrywide civil disobedience movement would be launched.
Select the correct answer using the codes given below. Show Answer


Q423) Consider the following statements:
1. A special session was held in Karachi to endorse the Gandhi-Irwin Pact reiterating The goal of Purna swaraj .
2. Two resolutions were adopted-one on Fundamental rights and other on National Economic program.
3. This was the first time the congress spelt out what swaraj would mean for the masses.
Which of the statements given above is/are correct? Show Answer


Q424) Consider the following statements:
1. In March 1931, the Gandhi-Irwin Pact was signed, and the government agreed to set all political prisoners free
2. In return, Gandhi agreed to discontinue the civil disobedience movement and participate as the sole representative of the Congress in the second Round Table Conference, which was held in London in September 1931.
Which of the statements given above is/are correct? Show Answer


Q425) The Government of India Act 1935, the voluminous and final constitutional effort at governing British India, articulated three major goals:
1. Establishing a loose federal structure,
2. Achieving provincial autonomy, and
3. Safeguarding minority interests through separate electorates.
Which of the statements given above is/are correct? Show Answer


Q426) Consider the following statements -
1. French East India Company In the middle of the seventeenth century Louis XIV's finance minister Colbert formed a French East India Company named Compagnie des Indes Orientales in 1664.
2. Louis XIV provided the Company with an interest-free loan of 3 million livre.
3. After initial attempts made to colonise Madagascar had proved a failure, the Company undertook a fresh expedition in 1667 under the command of Francis Caron accompanied by Marcara, a native of Ispahan reached India and set up the first French factory at Surat in 1668.
4. The second factory was set up at Masulipatnam in 1669. A factory was also developed at Chandernagar (Bengal) between 1690 and 1692.
5. Two major French trading posts in India were Pondicherry, established in 1674 by Francis martin, and Chandernagar.
Select the correct answer using the codes given below. Show Answer


Q427) Consider the following statements-
1. Before the East India Company established trade in India, John Mildenhall, a merchant adventurer, was the first Englishman who arrived in India in 1599 by the over land route, ostensibly for purpose of trade with Indian merchants.
2. On 31st December, 1600, Queen Elizabeth granted a Charter to the Company named 'The Governor and Company of Merchants of London Trading in the East Indies' the right to carry on trade with all countries of the East.
Which of the statements given above is/are correct? Show Answer


Q428) Consider the following statements-
1. The Constituent Assembly completed the work of drafting the constitution on 26 November 1949; on 26 January 1950, the Republic of India was officially proclaimed.
2. The Constituent Assembly elected Dr. Rajendra Prasad as the first President of India, taking over from Governor General Rajgopalachari.
3. Subsequently India invaded and annexed Goa and Portugal's other Indian enclaves in 1961), the French ceded Chandernagore in 1951, and Pondichéry and its remaining Indian colonies in 1956, and Sikkim voted to join the Indian Union in 1975.
Select the correct answer using the codes given below. Show Answer


Q429) Consider the following statements-
1. Gandhism designates the ideas and principles Gandhi promoted.
2. Of central importance is nonviolent resistance.
3. A Gandhian can mean either an individual who follows, or a specific philosophy which is attributed to, Gandhism.
Select the correct answer using the codes given below. Show Answer


Q430) What was/were the object/objects of Queen Victoria's Proclamation (1858)?
1. To disclaim any intention to annex Indian States
2. To place the Indian administration under the British Crown
3. To regulate East India Company's trade with India
Select the correct answer using the code given below Show Answer


Q431) The national motto of India, Satyameva Jayate’ inscribed below the Emblem of India is taken from Show Answer


Q432) Consider the following statements
1. High-yielding wheat was first introduced to India in 1968 by American agronomist Norman Borlaug. Borlaug has been hailed as the Father of the Green Revolution but M.S. Swaminathan is known as the "Father of the Green Revolution in India".
2. The methods adopted included the use of high yielding varieties (HYV) of seeds.
3. The production of wheat has produced the best results in fueling self-sufficiency of India.
4. Along with high yielding seeds and irrigation facilities, the enthusiasm of farmers mobilized the idea of agricultural revolution and is also credited to M. S. Swaminathan and his team had contributed towards the success of green revolution.
5. Due to the rise in use of chemical pesticides and fertilizers there were many negative effects on the soil and the land such as land degradation.
Select the correct answer using the codes given below. Show Answer


Q433) This was a strong indicator of the Indian people's support for complete Independence.
1. During the Second World War started, Viceroy Linlithgow unilaterally declared India a belligerent on the side of Britain, without consulting the elected Indian representatives.
2. In opposition to Linlithgow's action, the entire Congress leadership resigned from the local government councils.
3. many wanted to support the British war effort, and indeed the British Indian Army is the largest volunteer force, numbering 2,500,000 men during the war.
Select the correct answer using the codes given below. Show Answer


Q434) The independence movement saw the rise of movement/s:
1. The first of these, the kakori conspiracy (9 august 1925) was led by indian youth under the leadership of Pandit Ram Prasad Bismil;
2. Second was the azad hind movement led by netaji Subhas Chandra Bose which saw its inception early in the war and joined Germany and Japan to fight Britain;
3. The third one saw its inception in august 1942, was led by Lal Bahadur Shastri and reflected the common man resulting the failure of the cripps' mission to reach a consensus with the indian political leadership over the transfer of power after the war.
Select the correct answer using the codes given below. Show Answer


Q435) Consider the following statements :
1. The Quit India Movement (Bharat Chhodo Andolan) or the August Movement was a civil disobedience movement in India launched on 8 August 1942 in response to Gandhi's call for immediate independence of India and against sending Indians to World War II.
2. At the outbreak of war, the Congress Party had during the Wardha meeting of the working-committee in September 1939, passed a resolution conditionally supporting the fight against fascism, but were rebuffed when they asked for independence in return.
3. In March 1942, faced with an increasingly dissatisfied sub-continent only reluctantly participating in the war, and deteriorations in the war situation in Europe and South East Asia, and with growing dissatisfactions among Indian troops- especially in Europe- and among the civilian population in the sub-continent, the British government sent a delegation to India under Stafford Cripps, in what came to be known as the Cripps' Mission.
Select the correct answer using the codes given below. Show Answer


Q436) Consider the following statements
1. The aim of the Quit India movement was to bring the British Government to the negotiating table by holding the Allied War Effort hostage.
2. The call for determined but passive resistance that signified the certitude that Gandhi foresaw for the movement is best described by his call to Do or Die, issued on 8 August at the Gowalia Tank Maidan in Bombay, since renamed August Kranti Maidan (August Revolution Ground).
Select the correct answer using the codes given below. Show Answer


Q437) Consider the following statements
1. After two Japanese attacks on Christmas Island in late February and early March 1942, relations between the British officers and their Indian troops broke down.
2. On the night of 10 March, the Indian troops led by a Sikh policemen mutinied killing the five British soldiers and the imprisoning of the remaining 21 Europeans on the island.
3. on 31 March, a Japanese fleet arrived at the island and the Indians surrendered.
Select the correct answer using the codes given below. Show Answer


Q438) Consider the following statements
1. On 3 June 1947, Viscount Louis Mountbatten, the last British Governor-General of India, announced the partitioning of British India into India and Pakistan.
2. With the speedy passage through the British Parliament of the Indian Independence Act 1947, at 11:57 on 14 August 1947 Pakistan was declared a separate nation, and at 12:02, just after midnight, on 15 August 1947, India also became an independent nation.
3. Prime Minister Nehru and deputy prime minister Sardar Vallabhbhai Patel invited Mountbatten to continue as Governor General of India replacing in June 1948 by Chakravarti Rajagopalachari.
Select the correct answer using the codes given below. Show Answer


Q439) Rowlatt Acts, (February 1919), were passed by the Imperial Legislative Council, the legislature of British India. Consider the following statements with reference to the provisions/object of the Acts.
1. This act authorized the government to imprison for a maximum period of two years, without trial, any person suspected of terrorism.
2. The act provided speedy trial of the offenses by a special cell that consisted of 3 High Court Judges with no court of appeal above that panel.
3. This panel could also accept the evidences which were acceptable in the Indian Evidences Act.
4. The object was to replace the repressive provisions of the wartime Defence of India Act (1915) by a permanent law.
Select the correct answer using the codes given below: Show Answer


Q440) Consider the following statements about land revenue system during British time
1. The condition of temporary and sub-tenants was adhoc and they were subject to ruthless exploitation.
2. Frequent enhancement of rent, eviction on petty grounds and began (free service) were some of the prevalent ways of exploitation.
3. In India about 20 per cent of the agricultural land is devoted to share-cropping (bataz), where 50 per cent of the produce is the normal rent.
4. On several occasions, the peasant have to forego even three-fourths of the produce as rent.
Select the correct answer using the codes given below: Show Answer


Q441) Consider the following statements regarding British Queen Victoria's Proclamation in 1858 on Indian administration?
1. To disclaim any intention to annex Indian States
2. To place the Indian administration under the British Crown
3. To regulate East India Company's trade with India
Select the correct answer using the code given below. Show Answer


Q442) Brahmo Samaj was established by Raja Rammohan Roy and Arya Samaj was established by Swami Dayananda Saraswati. These two played an important role in the socio religious reforms of the country. Which of the following statements clearly bring out the differences between them?
1. Brahmo Samaj denied the infallibility of the Vedas while Arya Samaj accepted the supremacy of the Vedas.
2. While Arya Samaj advocated western ideas, the Brahmo Samaj was not open to western ideas.
3. While Hinduism was protected by Arya Samaj from hostile criticism of the missionaries, the Brahmo Samaj criticised Hinduism.
4. While the Brahmo Samaj gave the slogan of 'India for Indians' the Arya Samaj gave the slogan of 'Go Back to Vedas'.
Select the correct answer from the codes given below: Show Answer


Q443) The concept of Satyagraha devised by Mahatma Gandhi had been used in a number of instances during the freedom struggle. In this regard, consider the following statements:
1. The Champaran Satyagraha was against the introduction of Permanent Settlement system in Bihar in place of Tinkathia system.
2. The kheda Satyagraha was against the revenue policy of the Government.
3. The Bardoli Satyagraha was against revenue policy of the Bombay Government.
Which of the statements given above is/are correct? Show Answer


Q444) Consider the following statements regarding the Benoy-Badal-Dinesh Martyrdom:
1. Benoy, Badal and Dinesh assassinated Colonel N.S. Simpson, Inspector General of Prisons on December o8, 1930.
2. The Dalhousie Square in Calcutta has been renamed as 'B.B.D. Bagh' in the honour of these three Martyrs.
Which of the statements given above is/are correct? Show Answer


Q445) Consider these statements with respect to British policy of Carrot and Stick post-Swadeshi Movement:
1. Extremist were to be repressed.
2. Moderates were given some concessions on condition that they would alienate extremists.
3. Suppress both factions.
4. Suppress extremists and ignore moderates.
Select the correct statement(s) using the codes given below: Show Answer


Q446) Which among the following is a measure of Lord Curzon, intended to demoralize moderates, yet ended up revitalizing them:
1. Reconstitution of Calcutta corporation through the Calcutta Municipal Amendment Act of 1899 which reduced the number of elected representatives in it.
2. The Indian Universities Act of 1904 which placed the Calcutta University under complete government control.
3. Indian Official Secrets Amendment Act of 1904 which further restricted the press freedom.
4. Partition of Bengal in 1905 designed to weaken the Bengali nationalist.
Select the correct answer using the codes given below: Show Answer


Q447) The Ghadar Movement was channelized more out of enthusiasm than strategy”. In reference to the quote consider these statements:
1. Taken by surprise by the outbreak of the war and excited by the Kamagata Maru incident they sounded the bugles of war without examining the state of their army.
2. They underestimated the strength of British in India and let themselves imagine that a call of revolt to masses would strike fatal blow to British rule.
3. The movement failed to generate an effective and sustained leadership.
4. The Ghadar Movement was sustained more by the enthusiasm of the militants than their effective organization.
Select the correct statement(s) using the codes given below: Show Answer


Q448) In the consolidation of the British Empire in India, the annexation of Sindh had great significance. which of the following were the reasons behind annexation of Sindh by the British?
1. British fear of expansion of Russia to the east.
2. Commercial possibilities of the Indus river.
3. British desire to increase its influence in Persia and Afghanistan.
Select the correct answer from the codes given below: Show Answer


Q449) Assertion (A): Though Home Rule League could not provide any immediate results, it succeeded in serving the foundation for later movements.
Reason (R): Since there was no Pan India nationalistic effort at that point of time the Home Rule Movement provided a venting point for the budding nationalistic furore.
Select the correct answer using the codes given below: Show Answer


Q450) Which of the following statements clearly bring out the differences between the Permanent Settlement system and Ryotwari system?
1. While there was provision of 'pattas' in the Ryotwari system, it was not there in the Permanent Settlement system.
2. While the Ryotwari system was introduced in the Madras Presidency and Bombay Presidency, the Permanent Settlement system was introduced in Bengal, Bihar and Orissa.
3. While the ownership rights were made hereditary and transferable in the Permanent Settlement system, the ownership rights were not made hereditary in the Ryotwari settlement.
Select the correct answer from the codes given below: Show Answer


Q451) Assertion (A): Bhagat Singh came to believe that popular broad based movements alone could lead to a successful revolution.
Reason (R): Prior to his arrest in 1929, Bhagat Singh had abandoned his belief in terrorism.
Select the correct answer using the codes given below: Show Answer


Q452) Which among the following are the correct line of similarity between Swarajists and no-changers, which allowed them to accommodate each other:
1. Need of unity
2. However, useful parliamentary work to be the real sanctions to compel the government can be built by a mass movement
3. Essentiality of Gandhiji’s leadership
4. Fear of extremists taking over the movement
Select the correct answer using the codes given below: Show Answer


Q453) Which of these statements are correct in reference to the “Delhi Manifesto”?
1. It was conducted to decide the strategy prior to the Round Table Conference as acknowledged by Lord Irwin.
2. It was decided at the Conference to focus on when the dominion status be granted rather, if, it should be granted.
3. It was decided at the Conference to focus on formulation of the scheme to transfer dominion status to India.
4. It was conducted to decide whether to attend or boycott the Round Table Conference.
Select the correct answer using the codes given below: Show Answer


Q454) Assertion - The Congress stood idle when the August Award in 1932 allotted each minority a number of seats in the legislature to be elected on the basis of separate electorate.
Reason - Congress had agreed to an idea of separate electorate for Muslims in 1916 as a part of the compromise with Muslim League.
Select the correct answer using the codes given below: Show Answer


Q455) Which of the below statements are in consonance with the August offer of 1932:
1. The offer allotted each minority a number of seats in the legislatures to be elected on the basis of separate electorate.
2. The award separated depressed classes also as separate minority.
3. Gandhiji demanded that the representatives of depressed classes should be elected by general electorate under wide, if possible Universal Adult Franchise.
4. The disagreement over the August offer concluded with Gandhi-Irwin Pact.
Select the correct answer using the codes given below: Show Answer


Q456) Which among the following is not in consonance with the strategy of Congress Socialist Party?
1. Nationalism was a necessary stage on the way to socialism.
2. Socialists must work within the Congress ranks as it was the primary body leading the national struggle.
3. Socialists must give Congress and the national movement a left turn.
4. Socialists must organize the workers and peasants in their class organisations and make them wage struggle against their economic demands.
Select the correct answer using the codes given below: Show Answer


Q457) Which among the following reflect the impact of the left on National Movement
1. Resolution on Fundamental Rights and Economic Policy passed at Karachi session in 1931.
2. The resolution on economic policy in 1936 at Faizpur session.
3. The election manifesto of Congress in 1936.
4. The setting up of national planning committee in 1938.
Select the correct answer using the codes given below: Show Answer


Q458) On the debate over the issue on what course should the National Movement take post-withdrawal of Civil Disobedience, consider these factional opinions:
1. Constructive work in villages which would lead to the consolidation of people’s power.
2. Elections and work in legislative councils to be utilized to keep up the political interest and morale of the people.
3. Continuation of non-constitutional mass movement.
4. A nationwide Satyagrah to be launched with a demand of complete independence.
Select the correct statement(s) using the codes given below: Show Answer


Q459) Which among the following is not in consonance with the Government of India Act of 1935?
1. Representatives of the state to the federal legislature were to be appointed directly by the princes.
2. The franchise was limited to one- sixth of the adults.
3. Defence and foreign affairs to remain outside federal legislature.
4. The Governors appointed by the British were to retain special powers.
Select the correct answer using the codes given below: Show Answer


Q460) Which of the following were the major recommendations of the Simon Commission?
1. Abolition of dyarchy
2. Extension of responsible government to the provinces
3. Establishment of a federation of British India and Princely States
4. Continuation of Communal Electorates
Select the correct answer from the codes given below: Show Answer


Q461) After forming ministries in July 1935 which among the following are in consonance with the reform measures taken by the Congress?
1. Bans on illegal political organisation such as Hindustan Seva Dal and Youth Leagues and on political books and journals were lifted.
2. In UP tenancy act was passed in Oct 1939 which gave all statutory tenants both in Agra and Oudh, full hereditary rights.
3. The left Congressmen including Communist and Congress socialists opposed limitations on freedom to strike and for laying down new complicated procedure for registration of trade unions.
4. Congress president Subhash Chandra Bose in 1938 appointed Planning Committee.
Select the correct answer using the codes given below: Show Answer


Q462) From the characterstics given below Identify the movement undertaken/launched by Mahatma Gandhi in pursuit of achieving the goal of Independence of India ?
1. The movement became the name of an epoch in the life of India and of Gandhiji.
2. It was negative enough to be peaceful but positive enough to be effective.
3. It entailed denial,renunciation, and self-discipline.
4. It was training for self rule
Which of the following is referred to in the above-mentioned statements? Show Answer


Q463) The Khilafat Movement,(1919-1920) was a movement of Indian Muslims, led by Muhammad Ali and Shaukat Ali, that demanded certain important things . What was not there in the list of demands ? Show Answer


Q464) Consider the following statements about the Satyagraha of Mahatma gandhi -
1. Mahatma Gandhi first forged the distinctive techniques of non-violent protest known as
Satyagraha in Champaran.
2. The aim of Satyagraha was to first promote harmony between religions,and alerting upper -caste Indians to their discriminatory treatment of low castes and women in India
Select the correct answer using the codes given below: Show Answer


Q465) Round Table Conferences” in London in November 1930, culminated in “Gandhi-Irwin Pact’, by the terms of which
1. More seats were reserved for Indian in Central Legislative Council
2. Civil disobedience called off
3. All prisoners released
4. Salt manufacture allowed along the coast
Select the correct answers using the codes given below: Show Answer


Q466) Identify Governor General/Viceroy of British India from the description given below -
1. He adopted a vigorous forward policy and waged wars extensively.
2. His aggressive and imperialist polices paved the way for the general of expansion of the British Empire.
3. The conditions in India when he assumed power posed a serious threat to the British administration.
To which governor general is the reference being made in the above-mentioned statements? Show Answer


Q467) “No provocation can possibly justify (the) brutal murder of men who had been rendered defenceless and who had virtually thrown themselves on the mercy of the mob.” which incident is this statement alluding to- Show Answer


Q468) Quit India Movement was launched by Mahatma Gandhi in 1942.Which of the following can be considered as major motivator/s behind his decision to take such an extreme step-
1. the failure of the Cripps Mission
2. the advances of the Japanese in South-East Asia
3. the general frustration with the British in India
Select the correct statement/ statements using the codes given below. Show Answer


Q469) Consider the following statements about historic significance of Salt March of Mahatma Gandhi in April 1930 -
1. It first brought Mahatma Gandhi to world attention.
2. It was the first nationalist activity in which women participated in large numbers.
3. It forced upon the British the realization that they would have to devolve some power to the Indians.
4. After this event/March Gandhiji was called Mahatma by the people of India
Select the correct answers using the codes given below: Show Answer


Q470) Assertion -The growth and development of Peasant movement was indissolubly linked with the national struggle for freedom.
Reason - With the experience of the split of 1942, the Kisan Movement found that if it diverged too far and too clearly from the path of National Movement it tended to loose its mass base, as well as create split within ranks of Congress leadership.
Select the correct answer using the codes given below: Show Answer


Q471) Which among the following are the correct ways and policies through which British encouraged communalism:
1. Consistent treatment of Hindus, Muslims and Sikhs as separate communities and socio-political entities which had little in common.
2. Official favour and patronage were extended to the communalist.
3. The communal press, persons and agitations were shown extraordinary tolerance.
4. Communal demands were readily accepted, thus politically strengthening the communal organizations and their hold over people.
Select the correct answer using the codes given below: Show Answer


Q472) As a part of his speech at Gwalior Tank Gandhiji gave specific instructions to different sections of people. What were those instructions?
1. Allegiance towards Congress was expected from the government servants, but not their resignation.
2. Soldiers were asked not to leave their post but refuse to fire on our people.
3. The people of the princely states were asked to declare that they are part of the Indian nation and they would accept the leadership of the prince if the latter cast their lot with people but not otherwise.
4. Students were to give up studies if they were sure that they could continue to remain firm till independence was achieved.
Select the correct answer using the codes given below: Show Answer


Q473) Subsidiary Alliance was a system devised by Lord Wellesley in 1798. All those who entered into such an alliance with the British had to accept certain terms and conditions:
1. The British would be responsible for protecting their ally from external and internal threats to their power.
2. In the territory of the ally, a British armed contingent would be stationed.
3. The ally would have to provide the resources for maintaining this contingent of the British.
Select the correct option from the codes given below: Show Answer


Q474) Consider the following statement:
“I have travelled across the length and breadth of India and I have not seen one person who is a beggar, who is a thief. Such wealth I have seen in this country, such high moral values, people of such calibre that I do not think we would ever conquer this country, unless we break the very backbone of this nation, which is her spiritual and cultural heritage, and therefore, I propose that we replace her old and ancient education system, her culture, for if the Indians think that all that is foreign and English is good and greater than their own, they will lose their self-esteem, their native culture and they will become what we want them, a truly.”
Who gave this statement on which the present system of education is also based: Show Answer


Q475) Consider the following derogatory remarks made by a British planner about India, which Indians idiolise:
1. “Every sort of black body doing every sort of thing.”
2. “But oh the people the scallywags. Awful faces, to me degenerate, very dark, very naked, and awful habits of hair dressings. The bulk of the faces merely loony…”
3. “The natives do not improve on acquaintance.”
4. “They are children without the charm of heaven, and there seems a lot of carnal pleasure. And the caste rule, rules out any broad line of Christianity.”
The person was: Show Answer


Q476) Consider the following statements:
1. The IIbert Bill was strongly opposed by the Indians as it allowed the Indian Judges and Magistrates to try British offenders in criminal cases only at the District level.
2. The Simon Commission was protested as it was an all non-native Commission.
3. The Rowlatt Act was protested as it violated the principles of natural justice.
4. Partition of Bengal was protested as it was on the ground of communal lines.
Which of the statement given above are correct? Show Answer


Q477) Consider the following statements with reference to Wood’s Despatch :
1. Institution of a regular system of scholarship to enable meritorious students to pursue the higher course of study.
2. Helping the educational institutions founded by private effort to obtain grants from government funds
3. Provision of moral and religious education as an important component of curriculum
Which of the above were the features of Wood’s Despatch? Show Answer